How can a planet have a deadly eclipse-like “spotlight”?












24














A solar eclipse occurs when the moon passes between the Earth and the sun. The result is a giant shadow that sweeps across the Earth's surface.



This world has the opposite phenomenon. Instead of a giant shadow, this world has a giant deadly "spotlight" sweep across its surface.



Must achieve these effects:




  1. Must produce enough thermal power to kill humans (who are not native) and some (or all) non-native animals efficiently enough that full exposure to the light is (near?) certain doom.

  2. Should have easily-observed warning signs allowing vulnerable creatures roughly one minute to find shelter. None of the lifeforms on the planet have developed its meteorology well enough to predict them except by visual/thermal observation.

  3. Must be natural/meteorological/astronomical in nature. (Nothing like a giant orbiting laser).

  4. Most or all plants, fungi, etc (non-animals) should be able to survive.


These would be a plus:




  1. The starlight should normally be white, but the spotlight should be red (best case), blue (next best) or orange (third best). Otherwise, just brighter.

  2. Should occur a few times per week in at least one area on the planet.

  3. Should not be precisely periodic. If two occurrences are 36 hours apart, the next one might be 34 hours after, or 50 hours after. There may be a complex pattern, but it shouldn't be plausible to figure out over the course of a couple of weeks.


A "spotlight" that covers part of the planet is preferred, but if being larger than the planet is easier, that may be acceptable.



Here are a few ideas I was toying with, but I'm not sure how realistic they are:




  1. There are multiple small (or distant) stars in the system, each white in color, and multiple red moons surrounding the planet. The stars are clustered around the center of the system so there is still a day/night cycle (is this possible?) with a roughly Earthlike luminosity. The red moons reflect light on the surface more or less constantly but the intensity is relatively small compared to the normal light from the stars... Until enough moons reflect light from enough stars to the same spot on the planet's surface! I don't know if they can really reflect enough light for this to work though.

  2. Because of chaotic atmospheric conditions, atmospheric refraction causes the (single) star's rays to focus on a small area. I don't know if the color objective can be accomplished this way though.

  3. There is very thick cloud cover, but sometimes a hole opens that allows the full light of the star through onto part of the surface. Also not sure if the color objective can be accomplished.










share|improve this question




















  • 1




    Natural is strongly preferred.
    – Devsman
    2 days ago






  • 7




    I'm thinking something like a Dyson sphere around a very bright star, with one or more holes in it that periodically sweep over the planet. Normally the star appears to be very large and dim (as what you can see is actually the glowing outer surface of the Dyson sphere), but occasionally a hole rotates into position to scorch a path across the planet. It's technically artificial but could have been abandonened hundreds of millions of years earlier.
    – Gryphon
    2 days ago








  • 2




    Conservation of etendue. Can't have a moon that is brighter than its parent star using just ref*ction. The bigger issue is producing the spotlight effect though.
    – John Dvorak
    2 days ago






  • 13




    Scratch that. Devising anything that kills animals reliably while leaving plants untouched is pretty much impossible. For one thing, you'd have to explain why the animals don't just fancy a coat of lichen from day zero of their evolution.
    – John Dvorak
    2 days ago








  • 1




    @JohnDvorak Point taken. I'll update so it's not necessary for native animals to be killed.
    – Devsman
    2 days ago
















24














A solar eclipse occurs when the moon passes between the Earth and the sun. The result is a giant shadow that sweeps across the Earth's surface.



This world has the opposite phenomenon. Instead of a giant shadow, this world has a giant deadly "spotlight" sweep across its surface.



Must achieve these effects:




  1. Must produce enough thermal power to kill humans (who are not native) and some (or all) non-native animals efficiently enough that full exposure to the light is (near?) certain doom.

  2. Should have easily-observed warning signs allowing vulnerable creatures roughly one minute to find shelter. None of the lifeforms on the planet have developed its meteorology well enough to predict them except by visual/thermal observation.

  3. Must be natural/meteorological/astronomical in nature. (Nothing like a giant orbiting laser).

  4. Most or all plants, fungi, etc (non-animals) should be able to survive.


These would be a plus:




  1. The starlight should normally be white, but the spotlight should be red (best case), blue (next best) or orange (third best). Otherwise, just brighter.

  2. Should occur a few times per week in at least one area on the planet.

  3. Should not be precisely periodic. If two occurrences are 36 hours apart, the next one might be 34 hours after, or 50 hours after. There may be a complex pattern, but it shouldn't be plausible to figure out over the course of a couple of weeks.


A "spotlight" that covers part of the planet is preferred, but if being larger than the planet is easier, that may be acceptable.



Here are a few ideas I was toying with, but I'm not sure how realistic they are:




  1. There are multiple small (or distant) stars in the system, each white in color, and multiple red moons surrounding the planet. The stars are clustered around the center of the system so there is still a day/night cycle (is this possible?) with a roughly Earthlike luminosity. The red moons reflect light on the surface more or less constantly but the intensity is relatively small compared to the normal light from the stars... Until enough moons reflect light from enough stars to the same spot on the planet's surface! I don't know if they can really reflect enough light for this to work though.

  2. Because of chaotic atmospheric conditions, atmospheric refraction causes the (single) star's rays to focus on a small area. I don't know if the color objective can be accomplished this way though.

  3. There is very thick cloud cover, but sometimes a hole opens that allows the full light of the star through onto part of the surface. Also not sure if the color objective can be accomplished.










share|improve this question




















  • 1




    Natural is strongly preferred.
    – Devsman
    2 days ago






  • 7




    I'm thinking something like a Dyson sphere around a very bright star, with one or more holes in it that periodically sweep over the planet. Normally the star appears to be very large and dim (as what you can see is actually the glowing outer surface of the Dyson sphere), but occasionally a hole rotates into position to scorch a path across the planet. It's technically artificial but could have been abandonened hundreds of millions of years earlier.
    – Gryphon
    2 days ago








  • 2




    Conservation of etendue. Can't have a moon that is brighter than its parent star using just ref*ction. The bigger issue is producing the spotlight effect though.
    – John Dvorak
    2 days ago






  • 13




    Scratch that. Devising anything that kills animals reliably while leaving plants untouched is pretty much impossible. For one thing, you'd have to explain why the animals don't just fancy a coat of lichen from day zero of their evolution.
    – John Dvorak
    2 days ago








  • 1




    @JohnDvorak Point taken. I'll update so it's not necessary for native animals to be killed.
    – Devsman
    2 days ago














24












24








24


1





A solar eclipse occurs when the moon passes between the Earth and the sun. The result is a giant shadow that sweeps across the Earth's surface.



This world has the opposite phenomenon. Instead of a giant shadow, this world has a giant deadly "spotlight" sweep across its surface.



Must achieve these effects:




  1. Must produce enough thermal power to kill humans (who are not native) and some (or all) non-native animals efficiently enough that full exposure to the light is (near?) certain doom.

  2. Should have easily-observed warning signs allowing vulnerable creatures roughly one minute to find shelter. None of the lifeforms on the planet have developed its meteorology well enough to predict them except by visual/thermal observation.

  3. Must be natural/meteorological/astronomical in nature. (Nothing like a giant orbiting laser).

  4. Most or all plants, fungi, etc (non-animals) should be able to survive.


These would be a plus:




  1. The starlight should normally be white, but the spotlight should be red (best case), blue (next best) or orange (third best). Otherwise, just brighter.

  2. Should occur a few times per week in at least one area on the planet.

  3. Should not be precisely periodic. If two occurrences are 36 hours apart, the next one might be 34 hours after, or 50 hours after. There may be a complex pattern, but it shouldn't be plausible to figure out over the course of a couple of weeks.


A "spotlight" that covers part of the planet is preferred, but if being larger than the planet is easier, that may be acceptable.



Here are a few ideas I was toying with, but I'm not sure how realistic they are:




  1. There are multiple small (or distant) stars in the system, each white in color, and multiple red moons surrounding the planet. The stars are clustered around the center of the system so there is still a day/night cycle (is this possible?) with a roughly Earthlike luminosity. The red moons reflect light on the surface more or less constantly but the intensity is relatively small compared to the normal light from the stars... Until enough moons reflect light from enough stars to the same spot on the planet's surface! I don't know if they can really reflect enough light for this to work though.

  2. Because of chaotic atmospheric conditions, atmospheric refraction causes the (single) star's rays to focus on a small area. I don't know if the color objective can be accomplished this way though.

  3. There is very thick cloud cover, but sometimes a hole opens that allows the full light of the star through onto part of the surface. Also not sure if the color objective can be accomplished.










share|improve this question















A solar eclipse occurs when the moon passes between the Earth and the sun. The result is a giant shadow that sweeps across the Earth's surface.



This world has the opposite phenomenon. Instead of a giant shadow, this world has a giant deadly "spotlight" sweep across its surface.



Must achieve these effects:




  1. Must produce enough thermal power to kill humans (who are not native) and some (or all) non-native animals efficiently enough that full exposure to the light is (near?) certain doom.

  2. Should have easily-observed warning signs allowing vulnerable creatures roughly one minute to find shelter. None of the lifeforms on the planet have developed its meteorology well enough to predict them except by visual/thermal observation.

  3. Must be natural/meteorological/astronomical in nature. (Nothing like a giant orbiting laser).

  4. Most or all plants, fungi, etc (non-animals) should be able to survive.


These would be a plus:




  1. The starlight should normally be white, but the spotlight should be red (best case), blue (next best) or orange (third best). Otherwise, just brighter.

  2. Should occur a few times per week in at least one area on the planet.

  3. Should not be precisely periodic. If two occurrences are 36 hours apart, the next one might be 34 hours after, or 50 hours after. There may be a complex pattern, but it shouldn't be plausible to figure out over the course of a couple of weeks.


A "spotlight" that covers part of the planet is preferred, but if being larger than the planet is easier, that may be acceptable.



Here are a few ideas I was toying with, but I'm not sure how realistic they are:




  1. There are multiple small (or distant) stars in the system, each white in color, and multiple red moons surrounding the planet. The stars are clustered around the center of the system so there is still a day/night cycle (is this possible?) with a roughly Earthlike luminosity. The red moons reflect light on the surface more or less constantly but the intensity is relatively small compared to the normal light from the stars... Until enough moons reflect light from enough stars to the same spot on the planet's surface! I don't know if they can really reflect enough light for this to work though.

  2. Because of chaotic atmospheric conditions, atmospheric refraction causes the (single) star's rays to focus on a small area. I don't know if the color objective can be accomplished this way though.

  3. There is very thick cloud cover, but sometimes a hole opens that allows the full light of the star through onto part of the surface. Also not sure if the color objective can be accomplished.







astronomy weather






share|improve this question















share|improve this question













share|improve this question




share|improve this question








edited 2 days ago







Devsman

















asked 2 days ago









DevsmanDevsman

2,7751725




2,7751725








  • 1




    Natural is strongly preferred.
    – Devsman
    2 days ago






  • 7




    I'm thinking something like a Dyson sphere around a very bright star, with one or more holes in it that periodically sweep over the planet. Normally the star appears to be very large and dim (as what you can see is actually the glowing outer surface of the Dyson sphere), but occasionally a hole rotates into position to scorch a path across the planet. It's technically artificial but could have been abandonened hundreds of millions of years earlier.
    – Gryphon
    2 days ago








  • 2




    Conservation of etendue. Can't have a moon that is brighter than its parent star using just ref*ction. The bigger issue is producing the spotlight effect though.
    – John Dvorak
    2 days ago






  • 13




    Scratch that. Devising anything that kills animals reliably while leaving plants untouched is pretty much impossible. For one thing, you'd have to explain why the animals don't just fancy a coat of lichen from day zero of their evolution.
    – John Dvorak
    2 days ago








  • 1




    @JohnDvorak Point taken. I'll update so it's not necessary for native animals to be killed.
    – Devsman
    2 days ago














  • 1




    Natural is strongly preferred.
    – Devsman
    2 days ago






  • 7




    I'm thinking something like a Dyson sphere around a very bright star, with one or more holes in it that periodically sweep over the planet. Normally the star appears to be very large and dim (as what you can see is actually the glowing outer surface of the Dyson sphere), but occasionally a hole rotates into position to scorch a path across the planet. It's technically artificial but could have been abandonened hundreds of millions of years earlier.
    – Gryphon
    2 days ago








  • 2




    Conservation of etendue. Can't have a moon that is brighter than its parent star using just ref*ction. The bigger issue is producing the spotlight effect though.
    – John Dvorak
    2 days ago






  • 13




    Scratch that. Devising anything that kills animals reliably while leaving plants untouched is pretty much impossible. For one thing, you'd have to explain why the animals don't just fancy a coat of lichen from day zero of their evolution.
    – John Dvorak
    2 days ago








  • 1




    @JohnDvorak Point taken. I'll update so it's not necessary for native animals to be killed.
    – Devsman
    2 days ago








1




1




Natural is strongly preferred.
– Devsman
2 days ago




Natural is strongly preferred.
– Devsman
2 days ago




7




7




I'm thinking something like a Dyson sphere around a very bright star, with one or more holes in it that periodically sweep over the planet. Normally the star appears to be very large and dim (as what you can see is actually the glowing outer surface of the Dyson sphere), but occasionally a hole rotates into position to scorch a path across the planet. It's technically artificial but could have been abandonened hundreds of millions of years earlier.
– Gryphon
2 days ago






I'm thinking something like a Dyson sphere around a very bright star, with one or more holes in it that periodically sweep over the planet. Normally the star appears to be very large and dim (as what you can see is actually the glowing outer surface of the Dyson sphere), but occasionally a hole rotates into position to scorch a path across the planet. It's technically artificial but could have been abandonened hundreds of millions of years earlier.
– Gryphon
2 days ago






2




2




Conservation of etendue. Can't have a moon that is brighter than its parent star using just ref*ction. The bigger issue is producing the spotlight effect though.
– John Dvorak
2 days ago




Conservation of etendue. Can't have a moon that is brighter than its parent star using just ref*ction. The bigger issue is producing the spotlight effect though.
– John Dvorak
2 days ago




13




13




Scratch that. Devising anything that kills animals reliably while leaving plants untouched is pretty much impossible. For one thing, you'd have to explain why the animals don't just fancy a coat of lichen from day zero of their evolution.
– John Dvorak
2 days ago






Scratch that. Devising anything that kills animals reliably while leaving plants untouched is pretty much impossible. For one thing, you'd have to explain why the animals don't just fancy a coat of lichen from day zero of their evolution.
– John Dvorak
2 days ago






1




1




@JohnDvorak Point taken. I'll update so it's not necessary for native animals to be killed.
– Devsman
2 days ago




@JohnDvorak Point taken. I'll update so it's not necessary for native animals to be killed.
– Devsman
2 days ago










13 Answers
13






active

oldest

votes


















46














A wobbling pulsar will do the trick.



Pulsars emit a lot of energy in narrow beams that come from their poles. The slowest ones flash every few seconds; make its tilt wobble so that it is not pointing at the planet all the time. In addition, wobbling causes the pulsar to shoot at different points of the planet's orbit through time. The planet is hit when the pulsar's beams' path just happens to be passing by the planet.



If the pulsar is flashing every few milliseconds (as is normal for them), it will seem like a continuous beam for observers.



Finally, to make the beam small enough that it doesn't cover the entire planet and more, justify it with lensing from nearby nebulas, the planet's atmosphere, and maybe a black hole between the pulsar and the planet.






share|improve this answer



















  • 2




    Gah, I can't believe I didn't think of this. You got my +1.
    – Gryphon
    2 days ago






  • 6




    +1, even easier if it CAN cover the whole planet, which I think might work for the story.
    – Devsman
    2 days ago






  • 3




    @Renan Meaning the pulsar isn't the center of the system but just making a pass nearby? Yeah, that could work. en.wikipedia.org/wiki/Barnard%27s_Star#/media/…
    – Phil Frost
    2 days ago






  • 11




    Clarification (you might already understand this, but it is worth clarifying): pulsars don't "fire". They constantly emit their beam of light as long as the neutron star is accreting matter. The reason why they "blink" from our perspective is because the beam of light is focused (as you mention) and therefore only crosses our path once per rotation - not because it turns on and off. It's always on. The "period" is determined by its rotation rate, and doesn't have to be fast.
    – conman
    2 days ago






  • 2




    Make it a captured pulsar, meaning it went nova somewhere else and drifted through space until it was captured by the local start long enough ago in the past that local animals had some opportunity to adapt. Put it in an long outer orbit and put an asteroid belt between it and the habituated planet. Rotation of the pulsar and interference from the asteroid belt could cause an effect similar to what you are looking for.
    – Zack
    yesterday



















24














The remains of an ancient Dyson swarm



Not quite natural, but mostly non-technological. If a prior civilization had constructed a Dyson swarm around the system's star, the light coming from the star might be heavily occluded. Assume the sun is >10X hotter than ours (or the planet is much closer in), and there are enough collector bodies in the swarm to block some 90% of the sun's light at any time. If they are close enough in toward the sun, there will be enough diffraction around each collector that they wouldn't cast visible shadows, and could only be observed by direct observation of the sun, which requires a minimum level of technology to avoid blinding yourself.



The spotlight effect would occur when resonances in the orbital periods of the different bodies in the swarm cause gaps in coverage. The creating civilization could have arranged this purposefully to provide sunlight to further flung planets/stations, or be coincidental. The apparent brightness would grow gradually as more pieces of the swarm leave the "hole" in the field, so the warning sign would just be a rapid but gradual increase in brightness.



The spotlight color would probably be the same as the normal sunlight. However, if the star is very hot, heading toward blue spectrum, the swarm might occlude the blue/UV portion of the spectrum more and let redder light through (imagine if each collector is a giant solar array panel with no backing, e.g. microns of silicon). At the very least they would radiate heat in the infrared. If natives to the planet are used to these conditions, that might be their normal "white" light.






share|improve this answer

















  • 1




    Perhaps the gaps in coverage are caused by swarm members that have failed and fell out of orbit (shot down by a solar flare, by resonance with a heavy planet, collision with an in-falling comet...).
    – John Dvorak
    yesterday



















10














Any sort of passive light-focusing (with lenses, mirrors, etc) scheme is unlikely do more than to make slightly warm spots. The fundamental reason has to do with the conservation of etendue, and you can read more about it at Would a Moon made of water pose a threat to Earth during eclipses?



As such, if you want the spotlight to come from a moon, the moon would either need some kind of power source (which starts to sound like "giant lasers") or would need some natural mechanism to eject jets of energy or matter. As far as I know, all kinds of astrophysical jets would require something much more massive than a moon, so this seems like a dead-end.



I think the most feasible explanation is a planet which is ordinarily protected by its atmosphere and/or magnetosphere, but on occasion the weather aligns such that the protection is lost in an area. We to experience this to a small extent on earth: both the sun and earth have magnetic fields that vary over time. One trouble is if the earth's magnetosphere were periodically penetrated by the solar wind, the atmosphere would be stripped away. Though it could take a very long time -- perhaps it is interesting for your story to have a "dying planet".






share|improve this answer





















  • +1. If it enables the scenario, a dying planet is perfectly fine. It doesn't matter what happens to the planet after the story ends. :P
    – Devsman
    2 days ago



















4














Not entirely sure about the feasibility of this, but it's an idea I had when I read your question. I wonder if something like this would be possible through Gravitational Lensing. Essentially, this is when black holes (with enormous gravitational pulls) bend light around them, causing telescopic effects. I've linked the Wikipedia page for gravitational lensing as well as an article from Space.com below which you could read up on to give you a better idea of how it all works.



My idea though is that what if, just outside the edge of what can be seen from the planet, there's a system of black holes which pull light in such a manner that it's focussed into a thin beam, which cuts across the galaxy and occasionally burns its way across your planet? This would explain the huge intensity of the light as well as allowing a 'natural' explanation for how it's focussed so tightly.



Having it at such a distance would also mean that the appearance of the beam of light can't be predicted, as the people on the planet don't have the technology to either see that far into space, or understand what they're seeing. Besides, at such a distance that the black holes don't mess with the solar system's structure, the light would take very long to reach the planet. So when the beam lines up, its effects are only seen on the planet later (how much later depends on the distance. Centuries, or even millennia maybe).



As for the colour of the beam, we can assume the source of light is moving away from the black holes and the planet, which would cause Redshift. This would make a white light source look red to the observer. Frequency wise, we could assume that other objects in space (planets, dust clouds etc) often block the light from hitting our planet, but occasionally it slips through the gaps (like when you see the sun for a couple of seconds through a clearing in the clouds before it gets blocked again).



For non-animals to survive, perhaps they've evolved to feed on the huge light intensity and maybe even need it every few days to live? Or (depending on the history of the 'humans' on your planet) maybe everything else is evolved to survive the intense light to an extent, while humans aren't. Perhaps this is similar to how we have to wear clothes - we can't handle Earth's natural climates without external help. Maybe those caught outside of their radiation booths are killed rapidly, while those who stay inside are fine?



There's a lot of ways you can go with this idea and I think the rest is up to you. I've included a few links at the bottom you might be interested in.



Further Reading:



Gravitational lensing:



https://www.space.com/39999-how-gravitational-lenses-work.html



https://en.wikipedia.org/wiki/Gravitational_lens



Red and blue shift:



https://en.wikipedia.org/wiki/Redshift



https://www.space.com/25732-redshift-blueshift.html






share|improve this answer





















  • +1 for redshift
    – Devsman
    2 days ago






  • 8




    Sadly, the conservation of etendue makes this system of black hole lenses unworkable. If you're concentrating the light to a smaller output area, then it must also be distributed across a larger angle as it leaves the lens. XKCD's Randall Munroe did an excellent post on this topic: what-if.xkcd.com/145
    – Dubukay
    2 days ago






  • 1




    @Dubukay wow, never heard of that conservation law before. Thanks for bringing it to my attention! Love learning stuff on here haha
    – user43712
    2 days ago






  • 4




    However, if what we're going for is suspension-of-disbelief, make it a mini black hole in an inner orbit of the same system. Each time that BH passes between your unfortunate planet and the sun, lensing burns a scar right across the planet. If there's enough sunlight to start a fire with a magnifying glass, there's enough to do it through gravitational lensing (note that most of the planet will be nearly dark due to the lensing effect).
    – JBH
    2 days ago






  • 5




    Also, keep in mind that the mini black hole could also be one half of a binary star, in which case the orbits are preserved and the effect is the same, if a bit more complicated to calculate.
    – JBH
    2 days ago



















2














Ozone holes (https://en.wikipedia.org/wiki/Ozone_depletion) already cause real world health problems.



I think you could combine ozone holes and the loss of other atmospheric protection with the a coinciding local weakening of the magnetosphere. In the presence of a very energetic "sun", the results might be "deadly".



"Deadly" as in it might be immediately painful and certainly cancerous over time. Not deadly in the Vin Diesel kind of way you might be looking for though.






share|improve this answer





















  • A magnetosphere has no effect on electromagnetic radiation, and a breathable atmosphere would protect against solar wind.
    – Christopher James Huff
    2 days ago



















2














Let's have some cosmic fun.



Say that your planet orbits inside of an expanding red giant star.



Yep.



Now, say that someone or something decided to construct a shell around your world, perhaps before it had been engulfed by your host star as it evolved from the main sequence. This shell may be layers upon layers of orbital rings.



Now, say that this shell--made of some impossible material, probably--has some degree of translucency to it and the ability to change this translucency. You can have one hemisphere of the shell totally opaque to simulate night, with perhaps little points of translucency to simulate stars, and the other hemisphere translucent to simulate daylight (much less a simulation this point, because actual starlight would be entering).



Finally, let this shell have some circular region of total transparency which sweeps the planet, perhaps moving across the shell faster than the day-night cycle to give the folks on all sides, day or night, some of the action. You can think of the shell as being comprised of a bunch of little windows if you'd like, like pixels on a screen, that can be tinted and whatnot and can be made to filter out the unnecessary or harmful radiation of the star it is orbiting inside. The transparent portion would not filter these things, allowing death to beam down onto everything below. Heck, perhaps it even has little perforations or 'openings of the windows' that allow actual, high-energy particles to enter as well.






share|improve this answer



















  • 1




    "Say that your planet orbits inside of an expanding red giant star." It would quickly deorbit due to friction.
    – Renan
    2 days ago






  • 1




    @Renan Not as quickly as you'd think.
    – B.fox
    2 days ago










  • @B.fox, but still very quickly on a scale of planets. A very rough approximation, pretending that the classic drag equation is valid at all speeds, is that an Earth-sized planet will come to a complete stop after about 400 years.
    – Mark
    2 days ago












  • @Mark I'll have to follow up on that. I'm pretty sure the range was somewhere in the hundreds of thousands of years. The outer atmosphere of a red giant star is pretty diffuse, almost insubstantial.
    – B.fox
    yesterday












  • I'm not sure why this solution should depend on the planet being inside the sun. Take our own system, build such a shell around Mercury.
    – Mr Lister
    yesterday





















1














Constant lightning.



Consider a Rocheworld.
Can an atmosphere englobe a planetary ring?




Two tidally locked planets just outside the Roche Limit can orbit each
other and share a combined atmosphere. You would be able to fly from
one to the other without ever leaving the atmosphere and objects
placed at the lagrange points would be able to remain there.




These binary planets circle around each other. At one point in their orbit, their atmospheres touch (or you could have a moon graze the atmosphere of its planet). The friction of the two atmospheres against each other produces colossal amounts of electrical charge. When the two are close enough that the atmosphere can provide a path, charge can equilibrate across.



This takes the form of constant tremendous bolts of lightning that follow the path of the point where the two partners are closest to one another.






share|improve this answer





















  • I've never heard of a Rocheworld before... thats pretty cool!
    – Corbin Matheson
    17 hours ago



















1














I haven't seen this directly addressed, so I'll pose it as an answer:



TL;DR: Large moon with atmosphere refracts to a "point" on your planet. See below for etendue/thermodynamics, refraction, periodicity, and "warning signs".



A companion (moon, twin planet, or even planet as primary with your story set on a comfortable moon of a gas giant) large enough to hold a substantial atmosphere can perhaps be tuned to get the result you need.



Devise an atmosphere for the companion body with a powerful thermal inversion somewhere that reduces some of the spreading due to typical refraction of a density-stratified lens.



So we effectively have a ring-shaped lens, fairly narrow (edge view of the companion's atmosphere) but of very wide diameter (the companion itself), tuned to refract fairly well to a "point". The source of the light is the sun, and we will not get hotter than that. We do not need perfect point focus, but will gladly accept a central line of foci for various degrees of refraction, which generate -- you guessed it -- different colors of spotlight at different orbital distance of the large body from your planet's surface. Blue when it's close, red when it's far -- if it behaves like a proper lens-shaped lens. This would also result in color change as the effect sweeps from the edge of the home planet (farther) to the center (a bit closer). Warning signs would be similar to normal eclipses (the effect would only be observable from the very height of the eclipse). Finally, a combination of rotational and orbital planes for the three bodies involved can do wonders for making a simple periodic set of processes appear miserably non-periodic, particularly for observers located at different points on the surface of the home body.



I'll create a graphic. But I think this thing is doable with a lot less machinery than has been proposed so far, and without violating physics to the point of ridicule.






share|improve this answer



















  • 1




    The only reason this won't get ridiculed for violating physics is that most people don't know about conservation of etendue.
    – Mark
    2 days ago










  • Mark, how so? A garden variety magnifying glass does not get as hot as the point focused upon.
    – Haakon Dahl
    21 hours ago










  • Very, very simplified explanation of why "conservation of etendue" keeps this from working: the Sun is a small, very hot patch of sky surrounded by a whole lot of not-hot sky, so the ground doesn't normally get very hot. A magnifying glass heats things by creating a "virtual Sun" the size of the magnifying glass and exactly as hot as the Sun. This "virtual Sun" takes up a much larger part of the target's view of the sky, so the target gets hotter. Your lens planet, though, takes up only a small part of the view of the sky, and so doesn't do much more heating than the Sun, if that.
    – Mark
    9 hours ago



















0














The moon discussion referenced by @Phil Frost suggests part of the answer. A moon is too small so the lens-like body or phenomenon has to be big enough to cover all or most of the sky from the point of view of the target planet (which may itself be just a moon in a bigger system).



The problem is coming up with a celestial lens. If you can solve that, the rest is just a question of placing the target planet and the radiation source at a suitable scale and proximity.



A lens spotlight redirects light from a large area outside the "spot" so the first warning of the death ray's proximity would be a significant darkening, similar to a solar eclipse. In the distance you might see reflections from dust or clouds within the cone of concentrated light, so you can see if it's coming closer.






share|improve this answer








New contributor




maxwellsdemon is a new contributor to this site. Take care in asking for clarification, commenting, and answering.
Check out our Code of Conduct.


















  • You would also see the rim of the magnified sun start to appear from one side as the danger area approached, and could tell if it was going to pass you by or go right over you. But focusing light in such a way requires arrangements of matter too specifically contrived for nature to be a believable explanation. Your best bet would seem to be along the lines of neglectful precursors.
    – Christopher James Huff
    2 days ago



















0














The system could be a binary system with a neutron star or black hole orbiting close to the main star but in an eccentric orbit that lasts a few days. When it draws close to the primary it pulls off huge masses of coronal gasses and causes massive incredibly intense solar flares. If these incidents happen at the same time as the planet is in the wrong part of the sky then you can expect some serious pyrotechnics to hit the day side of the planet for a few hours.



The black hole/neutron star would most likely have been captured rather than be an original part of the system, explaining the eccentric orbit and any unusual spin needed etc.



It doesn't take a big stretch to somehow say that the gravity and magnetic fields of the neutron star focuses the ejections into beams somehow. So every X hours you get massive beams of solar energy being fired in random directions. You can then get the variance by saying whether those beams hit your planet or not.






share|improve this answer

















  • 1




    I think this is too deadly. The situation you describe is similar to a classic nova, and the 10,000-fold increase in solar output from one of those will quite handily sterilize a planet.
    – Mark
    yesterday



















-1














A small moon-like object in the planet+star's L1 Lagrange point (i.e., the point where the star's and planet's gravity exactly cancel out) would do the trick. Thanks to the wave nature of light, the moon will generate an Arago spot (bright spot) on the surface of your planet. Pick a small and bright star, perhaps a young white dwarf or neutron star.



To have a moving spot, the moon would need to move about a bit. You can think of an orbit around the L1 point. This should not be too hard.



Much more difficult is the fact that the L1 point is an unstable equilibrium point. Objects do not remain in orbit around the L1 point looking only at gravitational forces. Here, some handwaving is necessary. Perhaps the pressure from stellar winds from the central star have some stabilising influence. Perhaps heating of parts of the planets not obscured by the moon will cause massive out-gassing of the oceans into space, providing a stabilising pressure.



Regardless, it's definitely not a predictable situation, which should make it ideal for your story.






share|improve this answer





















  • This doesn't work for two major reasons. First, you won't get an Arago spot because the Sun isn't even remotely like a point source. And second, even if you did get an Arago spot, it would only be as bright (and as dangerous) as direct sunlight.
    – Mark
    9 hours ago



















-2














Reverse Solar Eclipse.

An orbiting Ocean, say a liquid helium moon of the correct size. Meaning, blocking the entire star, or larger. It could create a focusing and penumbra effect based on your need for destruction. Reinforce your physics.






share|improve this answer










New contributor




SiGGER is a new contributor to this site. Take care in asking for clarification, commenting, and answering.
Check out our Code of Conduct.














  • 2




    The correct size is roughly "fills the entire sky". See conservation of etendue, XKCD What-If #145, and the answers to this question.
    – Mark
    2 days ago



















-3














A transparent sphere works as a burning glass so a moon of (impossible) clear material should do the trick by concentrating the rays from the sun if it orbited at the right distance.



Trouble is that absorbtion would eat most of the light if the diameter was more than a kilometer. A thin ice-shell might work, but good luck with explaining the origin (and stability!!) of that ;-)






share|improve this answer








New contributor




Mads Horn is a new contributor to this site. Take care in asking for clarification, commenting, and answering.
Check out our Code of Conduct.


















  • Welcome to Worldbuilding, Mads Horn! If you have a moment, please take the tour and visit the help center to learn more about the site. You may also find Worldbuilding Meta and The Sandbox useful. Here is a meta post on the culture and style of Worldbuilding.SE, just to help you understand our scope and methods, and how we do things here. Have fun!
    – Gryphon
    2 days ago






  • 1




    A thin layer of ice may let enough light through, but it won't have any significant lensing effect, not to mention it wouldn't survive without collapsing spectacularly for more than a few days after being conjured at best, let alone form naturally in the first place.
    – John Dvorak
    2 days ago






  • 1




    "If it orbited at the right distance": the focal distance of a ball lens is $f = nD / 4(n - 1)$, with n being the index of refraction of the material and D the diameter of the sphere. For glass, this works out at about 0.8 D, so that orbit must be very close to the surface. Not to mention that the focus lies on the optical axis, so that it wont fall on the surface unless the moon is in conjuction with the Sun. And ball lenses are horrible lenses, they won't focus the light in a nice focal spot.
    – AlexP
    2 days ago








  • 6




    Won't work, for reasons more than just absorption. See Would a Moon made of water pose a threat to Earth during eclipses?
    – Phil Frost
    2 days ago













Your Answer





StackExchange.ifUsing("editor", function () {
return StackExchange.using("mathjaxEditing", function () {
StackExchange.MarkdownEditor.creationCallbacks.add(function (editor, postfix) {
StackExchange.mathjaxEditing.prepareWmdForMathJax(editor, postfix, [["$", "$"], ["\\(","\\)"]]);
});
});
}, "mathjax-editing");

StackExchange.ready(function() {
var channelOptions = {
tags: "".split(" "),
id: "579"
};
initTagRenderer("".split(" "), "".split(" "), channelOptions);

StackExchange.using("externalEditor", function() {
// Have to fire editor after snippets, if snippets enabled
if (StackExchange.settings.snippets.snippetsEnabled) {
StackExchange.using("snippets", function() {
createEditor();
});
}
else {
createEditor();
}
});

function createEditor() {
StackExchange.prepareEditor({
heartbeatType: 'answer',
autoActivateHeartbeat: false,
convertImagesToLinks: false,
noModals: true,
showLowRepImageUploadWarning: true,
reputationToPostImages: null,
bindNavPrevention: true,
postfix: "",
imageUploader: {
brandingHtml: "Powered by u003ca class="icon-imgur-white" href="https://imgur.com/"u003eu003c/au003e",
contentPolicyHtml: "User contributions licensed under u003ca href="https://creativecommons.org/licenses/by-sa/3.0/"u003ecc by-sa 3.0 with attribution requiredu003c/au003e u003ca href="https://stackoverflow.com/legal/content-policy"u003e(content policy)u003c/au003e",
allowUrls: true
},
noCode: true, onDemand: true,
discardSelector: ".discard-answer"
,immediatelyShowMarkdownHelp:true
});


}
});














draft saved

draft discarded


















StackExchange.ready(
function () {
StackExchange.openid.initPostLogin('.new-post-login', 'https%3a%2f%2fworldbuilding.stackexchange.com%2fquestions%2f135840%2fhow-can-a-planet-have-a-deadly-eclipse-like-spotlight%23new-answer', 'question_page');
}
);

Post as a guest















Required, but never shown

























13 Answers
13






active

oldest

votes








13 Answers
13






active

oldest

votes









active

oldest

votes






active

oldest

votes









46














A wobbling pulsar will do the trick.



Pulsars emit a lot of energy in narrow beams that come from their poles. The slowest ones flash every few seconds; make its tilt wobble so that it is not pointing at the planet all the time. In addition, wobbling causes the pulsar to shoot at different points of the planet's orbit through time. The planet is hit when the pulsar's beams' path just happens to be passing by the planet.



If the pulsar is flashing every few milliseconds (as is normal for them), it will seem like a continuous beam for observers.



Finally, to make the beam small enough that it doesn't cover the entire planet and more, justify it with lensing from nearby nebulas, the planet's atmosphere, and maybe a black hole between the pulsar and the planet.






share|improve this answer



















  • 2




    Gah, I can't believe I didn't think of this. You got my +1.
    – Gryphon
    2 days ago






  • 6




    +1, even easier if it CAN cover the whole planet, which I think might work for the story.
    – Devsman
    2 days ago






  • 3




    @Renan Meaning the pulsar isn't the center of the system but just making a pass nearby? Yeah, that could work. en.wikipedia.org/wiki/Barnard%27s_Star#/media/…
    – Phil Frost
    2 days ago






  • 11




    Clarification (you might already understand this, but it is worth clarifying): pulsars don't "fire". They constantly emit their beam of light as long as the neutron star is accreting matter. The reason why they "blink" from our perspective is because the beam of light is focused (as you mention) and therefore only crosses our path once per rotation - not because it turns on and off. It's always on. The "period" is determined by its rotation rate, and doesn't have to be fast.
    – conman
    2 days ago






  • 2




    Make it a captured pulsar, meaning it went nova somewhere else and drifted through space until it was captured by the local start long enough ago in the past that local animals had some opportunity to adapt. Put it in an long outer orbit and put an asteroid belt between it and the habituated planet. Rotation of the pulsar and interference from the asteroid belt could cause an effect similar to what you are looking for.
    – Zack
    yesterday
















46














A wobbling pulsar will do the trick.



Pulsars emit a lot of energy in narrow beams that come from their poles. The slowest ones flash every few seconds; make its tilt wobble so that it is not pointing at the planet all the time. In addition, wobbling causes the pulsar to shoot at different points of the planet's orbit through time. The planet is hit when the pulsar's beams' path just happens to be passing by the planet.



If the pulsar is flashing every few milliseconds (as is normal for them), it will seem like a continuous beam for observers.



Finally, to make the beam small enough that it doesn't cover the entire planet and more, justify it with lensing from nearby nebulas, the planet's atmosphere, and maybe a black hole between the pulsar and the planet.






share|improve this answer



















  • 2




    Gah, I can't believe I didn't think of this. You got my +1.
    – Gryphon
    2 days ago






  • 6




    +1, even easier if it CAN cover the whole planet, which I think might work for the story.
    – Devsman
    2 days ago






  • 3




    @Renan Meaning the pulsar isn't the center of the system but just making a pass nearby? Yeah, that could work. en.wikipedia.org/wiki/Barnard%27s_Star#/media/…
    – Phil Frost
    2 days ago






  • 11




    Clarification (you might already understand this, but it is worth clarifying): pulsars don't "fire". They constantly emit their beam of light as long as the neutron star is accreting matter. The reason why they "blink" from our perspective is because the beam of light is focused (as you mention) and therefore only crosses our path once per rotation - not because it turns on and off. It's always on. The "period" is determined by its rotation rate, and doesn't have to be fast.
    – conman
    2 days ago






  • 2




    Make it a captured pulsar, meaning it went nova somewhere else and drifted through space until it was captured by the local start long enough ago in the past that local animals had some opportunity to adapt. Put it in an long outer orbit and put an asteroid belt between it and the habituated planet. Rotation of the pulsar and interference from the asteroid belt could cause an effect similar to what you are looking for.
    – Zack
    yesterday














46












46








46






A wobbling pulsar will do the trick.



Pulsars emit a lot of energy in narrow beams that come from their poles. The slowest ones flash every few seconds; make its tilt wobble so that it is not pointing at the planet all the time. In addition, wobbling causes the pulsar to shoot at different points of the planet's orbit through time. The planet is hit when the pulsar's beams' path just happens to be passing by the planet.



If the pulsar is flashing every few milliseconds (as is normal for them), it will seem like a continuous beam for observers.



Finally, to make the beam small enough that it doesn't cover the entire planet and more, justify it with lensing from nearby nebulas, the planet's atmosphere, and maybe a black hole between the pulsar and the planet.






share|improve this answer














A wobbling pulsar will do the trick.



Pulsars emit a lot of energy in narrow beams that come from their poles. The slowest ones flash every few seconds; make its tilt wobble so that it is not pointing at the planet all the time. In addition, wobbling causes the pulsar to shoot at different points of the planet's orbit through time. The planet is hit when the pulsar's beams' path just happens to be passing by the planet.



If the pulsar is flashing every few milliseconds (as is normal for them), it will seem like a continuous beam for observers.



Finally, to make the beam small enough that it doesn't cover the entire planet and more, justify it with lensing from nearby nebulas, the planet's atmosphere, and maybe a black hole between the pulsar and the planet.







share|improve this answer














share|improve this answer



share|improve this answer








edited yesterday

























answered 2 days ago









RenanRenan

44.3k11100226




44.3k11100226








  • 2




    Gah, I can't believe I didn't think of this. You got my +1.
    – Gryphon
    2 days ago






  • 6




    +1, even easier if it CAN cover the whole planet, which I think might work for the story.
    – Devsman
    2 days ago






  • 3




    @Renan Meaning the pulsar isn't the center of the system but just making a pass nearby? Yeah, that could work. en.wikipedia.org/wiki/Barnard%27s_Star#/media/…
    – Phil Frost
    2 days ago






  • 11




    Clarification (you might already understand this, but it is worth clarifying): pulsars don't "fire". They constantly emit their beam of light as long as the neutron star is accreting matter. The reason why they "blink" from our perspective is because the beam of light is focused (as you mention) and therefore only crosses our path once per rotation - not because it turns on and off. It's always on. The "period" is determined by its rotation rate, and doesn't have to be fast.
    – conman
    2 days ago






  • 2




    Make it a captured pulsar, meaning it went nova somewhere else and drifted through space until it was captured by the local start long enough ago in the past that local animals had some opportunity to adapt. Put it in an long outer orbit and put an asteroid belt between it and the habituated planet. Rotation of the pulsar and interference from the asteroid belt could cause an effect similar to what you are looking for.
    – Zack
    yesterday














  • 2




    Gah, I can't believe I didn't think of this. You got my +1.
    – Gryphon
    2 days ago






  • 6




    +1, even easier if it CAN cover the whole planet, which I think might work for the story.
    – Devsman
    2 days ago






  • 3




    @Renan Meaning the pulsar isn't the center of the system but just making a pass nearby? Yeah, that could work. en.wikipedia.org/wiki/Barnard%27s_Star#/media/…
    – Phil Frost
    2 days ago






  • 11




    Clarification (you might already understand this, but it is worth clarifying): pulsars don't "fire". They constantly emit their beam of light as long as the neutron star is accreting matter. The reason why they "blink" from our perspective is because the beam of light is focused (as you mention) and therefore only crosses our path once per rotation - not because it turns on and off. It's always on. The "period" is determined by its rotation rate, and doesn't have to be fast.
    – conman
    2 days ago






  • 2




    Make it a captured pulsar, meaning it went nova somewhere else and drifted through space until it was captured by the local start long enough ago in the past that local animals had some opportunity to adapt. Put it in an long outer orbit and put an asteroid belt between it and the habituated planet. Rotation of the pulsar and interference from the asteroid belt could cause an effect similar to what you are looking for.
    – Zack
    yesterday








2




2




Gah, I can't believe I didn't think of this. You got my +1.
– Gryphon
2 days ago




Gah, I can't believe I didn't think of this. You got my +1.
– Gryphon
2 days ago




6




6




+1, even easier if it CAN cover the whole planet, which I think might work for the story.
– Devsman
2 days ago




+1, even easier if it CAN cover the whole planet, which I think might work for the story.
– Devsman
2 days ago




3




3




@Renan Meaning the pulsar isn't the center of the system but just making a pass nearby? Yeah, that could work. en.wikipedia.org/wiki/Barnard%27s_Star#/media/…
– Phil Frost
2 days ago




@Renan Meaning the pulsar isn't the center of the system but just making a pass nearby? Yeah, that could work. en.wikipedia.org/wiki/Barnard%27s_Star#/media/…
– Phil Frost
2 days ago




11




11




Clarification (you might already understand this, but it is worth clarifying): pulsars don't "fire". They constantly emit their beam of light as long as the neutron star is accreting matter. The reason why they "blink" from our perspective is because the beam of light is focused (as you mention) and therefore only crosses our path once per rotation - not because it turns on and off. It's always on. The "period" is determined by its rotation rate, and doesn't have to be fast.
– conman
2 days ago




Clarification (you might already understand this, but it is worth clarifying): pulsars don't "fire". They constantly emit their beam of light as long as the neutron star is accreting matter. The reason why they "blink" from our perspective is because the beam of light is focused (as you mention) and therefore only crosses our path once per rotation - not because it turns on and off. It's always on. The "period" is determined by its rotation rate, and doesn't have to be fast.
– conman
2 days ago




2




2




Make it a captured pulsar, meaning it went nova somewhere else and drifted through space until it was captured by the local start long enough ago in the past that local animals had some opportunity to adapt. Put it in an long outer orbit and put an asteroid belt between it and the habituated planet. Rotation of the pulsar and interference from the asteroid belt could cause an effect similar to what you are looking for.
– Zack
yesterday




Make it a captured pulsar, meaning it went nova somewhere else and drifted through space until it was captured by the local start long enough ago in the past that local animals had some opportunity to adapt. Put it in an long outer orbit and put an asteroid belt between it and the habituated planet. Rotation of the pulsar and interference from the asteroid belt could cause an effect similar to what you are looking for.
– Zack
yesterday











24














The remains of an ancient Dyson swarm



Not quite natural, but mostly non-technological. If a prior civilization had constructed a Dyson swarm around the system's star, the light coming from the star might be heavily occluded. Assume the sun is >10X hotter than ours (or the planet is much closer in), and there are enough collector bodies in the swarm to block some 90% of the sun's light at any time. If they are close enough in toward the sun, there will be enough diffraction around each collector that they wouldn't cast visible shadows, and could only be observed by direct observation of the sun, which requires a minimum level of technology to avoid blinding yourself.



The spotlight effect would occur when resonances in the orbital periods of the different bodies in the swarm cause gaps in coverage. The creating civilization could have arranged this purposefully to provide sunlight to further flung planets/stations, or be coincidental. The apparent brightness would grow gradually as more pieces of the swarm leave the "hole" in the field, so the warning sign would just be a rapid but gradual increase in brightness.



The spotlight color would probably be the same as the normal sunlight. However, if the star is very hot, heading toward blue spectrum, the swarm might occlude the blue/UV portion of the spectrum more and let redder light through (imagine if each collector is a giant solar array panel with no backing, e.g. microns of silicon). At the very least they would radiate heat in the infrared. If natives to the planet are used to these conditions, that might be their normal "white" light.






share|improve this answer

















  • 1




    Perhaps the gaps in coverage are caused by swarm members that have failed and fell out of orbit (shot down by a solar flare, by resonance with a heavy planet, collision with an in-falling comet...).
    – John Dvorak
    yesterday
















24














The remains of an ancient Dyson swarm



Not quite natural, but mostly non-technological. If a prior civilization had constructed a Dyson swarm around the system's star, the light coming from the star might be heavily occluded. Assume the sun is >10X hotter than ours (or the planet is much closer in), and there are enough collector bodies in the swarm to block some 90% of the sun's light at any time. If they are close enough in toward the sun, there will be enough diffraction around each collector that they wouldn't cast visible shadows, and could only be observed by direct observation of the sun, which requires a minimum level of technology to avoid blinding yourself.



The spotlight effect would occur when resonances in the orbital periods of the different bodies in the swarm cause gaps in coverage. The creating civilization could have arranged this purposefully to provide sunlight to further flung planets/stations, or be coincidental. The apparent brightness would grow gradually as more pieces of the swarm leave the "hole" in the field, so the warning sign would just be a rapid but gradual increase in brightness.



The spotlight color would probably be the same as the normal sunlight. However, if the star is very hot, heading toward blue spectrum, the swarm might occlude the blue/UV portion of the spectrum more and let redder light through (imagine if each collector is a giant solar array panel with no backing, e.g. microns of silicon). At the very least they would radiate heat in the infrared. If natives to the planet are used to these conditions, that might be their normal "white" light.






share|improve this answer

















  • 1




    Perhaps the gaps in coverage are caused by swarm members that have failed and fell out of orbit (shot down by a solar flare, by resonance with a heavy planet, collision with an in-falling comet...).
    – John Dvorak
    yesterday














24












24








24






The remains of an ancient Dyson swarm



Not quite natural, but mostly non-technological. If a prior civilization had constructed a Dyson swarm around the system's star, the light coming from the star might be heavily occluded. Assume the sun is >10X hotter than ours (or the planet is much closer in), and there are enough collector bodies in the swarm to block some 90% of the sun's light at any time. If they are close enough in toward the sun, there will be enough diffraction around each collector that they wouldn't cast visible shadows, and could only be observed by direct observation of the sun, which requires a minimum level of technology to avoid blinding yourself.



The spotlight effect would occur when resonances in the orbital periods of the different bodies in the swarm cause gaps in coverage. The creating civilization could have arranged this purposefully to provide sunlight to further flung planets/stations, or be coincidental. The apparent brightness would grow gradually as more pieces of the swarm leave the "hole" in the field, so the warning sign would just be a rapid but gradual increase in brightness.



The spotlight color would probably be the same as the normal sunlight. However, if the star is very hot, heading toward blue spectrum, the swarm might occlude the blue/UV portion of the spectrum more and let redder light through (imagine if each collector is a giant solar array panel with no backing, e.g. microns of silicon). At the very least they would radiate heat in the infrared. If natives to the planet are used to these conditions, that might be their normal "white" light.






share|improve this answer












The remains of an ancient Dyson swarm



Not quite natural, but mostly non-technological. If a prior civilization had constructed a Dyson swarm around the system's star, the light coming from the star might be heavily occluded. Assume the sun is >10X hotter than ours (or the planet is much closer in), and there are enough collector bodies in the swarm to block some 90% of the sun's light at any time. If they are close enough in toward the sun, there will be enough diffraction around each collector that they wouldn't cast visible shadows, and could only be observed by direct observation of the sun, which requires a minimum level of technology to avoid blinding yourself.



The spotlight effect would occur when resonances in the orbital periods of the different bodies in the swarm cause gaps in coverage. The creating civilization could have arranged this purposefully to provide sunlight to further flung planets/stations, or be coincidental. The apparent brightness would grow gradually as more pieces of the swarm leave the "hole" in the field, so the warning sign would just be a rapid but gradual increase in brightness.



The spotlight color would probably be the same as the normal sunlight. However, if the star is very hot, heading toward blue spectrum, the swarm might occlude the blue/UV portion of the spectrum more and let redder light through (imagine if each collector is a giant solar array panel with no backing, e.g. microns of silicon). At the very least they would radiate heat in the infrared. If natives to the planet are used to these conditions, that might be their normal "white" light.







share|improve this answer












share|improve this answer



share|improve this answer










answered 2 days ago









thegreatemuthegreatemu

1,180210




1,180210








  • 1




    Perhaps the gaps in coverage are caused by swarm members that have failed and fell out of orbit (shot down by a solar flare, by resonance with a heavy planet, collision with an in-falling comet...).
    – John Dvorak
    yesterday














  • 1




    Perhaps the gaps in coverage are caused by swarm members that have failed and fell out of orbit (shot down by a solar flare, by resonance with a heavy planet, collision with an in-falling comet...).
    – John Dvorak
    yesterday








1




1




Perhaps the gaps in coverage are caused by swarm members that have failed and fell out of orbit (shot down by a solar flare, by resonance with a heavy planet, collision with an in-falling comet...).
– John Dvorak
yesterday




Perhaps the gaps in coverage are caused by swarm members that have failed and fell out of orbit (shot down by a solar flare, by resonance with a heavy planet, collision with an in-falling comet...).
– John Dvorak
yesterday











10














Any sort of passive light-focusing (with lenses, mirrors, etc) scheme is unlikely do more than to make slightly warm spots. The fundamental reason has to do with the conservation of etendue, and you can read more about it at Would a Moon made of water pose a threat to Earth during eclipses?



As such, if you want the spotlight to come from a moon, the moon would either need some kind of power source (which starts to sound like "giant lasers") or would need some natural mechanism to eject jets of energy or matter. As far as I know, all kinds of astrophysical jets would require something much more massive than a moon, so this seems like a dead-end.



I think the most feasible explanation is a planet which is ordinarily protected by its atmosphere and/or magnetosphere, but on occasion the weather aligns such that the protection is lost in an area. We to experience this to a small extent on earth: both the sun and earth have magnetic fields that vary over time. One trouble is if the earth's magnetosphere were periodically penetrated by the solar wind, the atmosphere would be stripped away. Though it could take a very long time -- perhaps it is interesting for your story to have a "dying planet".






share|improve this answer





















  • +1. If it enables the scenario, a dying planet is perfectly fine. It doesn't matter what happens to the planet after the story ends. :P
    – Devsman
    2 days ago
















10














Any sort of passive light-focusing (with lenses, mirrors, etc) scheme is unlikely do more than to make slightly warm spots. The fundamental reason has to do with the conservation of etendue, and you can read more about it at Would a Moon made of water pose a threat to Earth during eclipses?



As such, if you want the spotlight to come from a moon, the moon would either need some kind of power source (which starts to sound like "giant lasers") or would need some natural mechanism to eject jets of energy or matter. As far as I know, all kinds of astrophysical jets would require something much more massive than a moon, so this seems like a dead-end.



I think the most feasible explanation is a planet which is ordinarily protected by its atmosphere and/or magnetosphere, but on occasion the weather aligns such that the protection is lost in an area. We to experience this to a small extent on earth: both the sun and earth have magnetic fields that vary over time. One trouble is if the earth's magnetosphere were periodically penetrated by the solar wind, the atmosphere would be stripped away. Though it could take a very long time -- perhaps it is interesting for your story to have a "dying planet".






share|improve this answer





















  • +1. If it enables the scenario, a dying planet is perfectly fine. It doesn't matter what happens to the planet after the story ends. :P
    – Devsman
    2 days ago














10












10








10






Any sort of passive light-focusing (with lenses, mirrors, etc) scheme is unlikely do more than to make slightly warm spots. The fundamental reason has to do with the conservation of etendue, and you can read more about it at Would a Moon made of water pose a threat to Earth during eclipses?



As such, if you want the spotlight to come from a moon, the moon would either need some kind of power source (which starts to sound like "giant lasers") or would need some natural mechanism to eject jets of energy or matter. As far as I know, all kinds of astrophysical jets would require something much more massive than a moon, so this seems like a dead-end.



I think the most feasible explanation is a planet which is ordinarily protected by its atmosphere and/or magnetosphere, but on occasion the weather aligns such that the protection is lost in an area. We to experience this to a small extent on earth: both the sun and earth have magnetic fields that vary over time. One trouble is if the earth's magnetosphere were periodically penetrated by the solar wind, the atmosphere would be stripped away. Though it could take a very long time -- perhaps it is interesting for your story to have a "dying planet".






share|improve this answer












Any sort of passive light-focusing (with lenses, mirrors, etc) scheme is unlikely do more than to make slightly warm spots. The fundamental reason has to do with the conservation of etendue, and you can read more about it at Would a Moon made of water pose a threat to Earth during eclipses?



As such, if you want the spotlight to come from a moon, the moon would either need some kind of power source (which starts to sound like "giant lasers") or would need some natural mechanism to eject jets of energy or matter. As far as I know, all kinds of astrophysical jets would require something much more massive than a moon, so this seems like a dead-end.



I think the most feasible explanation is a planet which is ordinarily protected by its atmosphere and/or magnetosphere, but on occasion the weather aligns such that the protection is lost in an area. We to experience this to a small extent on earth: both the sun and earth have magnetic fields that vary over time. One trouble is if the earth's magnetosphere were periodically penetrated by the solar wind, the atmosphere would be stripped away. Though it could take a very long time -- perhaps it is interesting for your story to have a "dying planet".







share|improve this answer












share|improve this answer



share|improve this answer










answered 2 days ago









Phil FrostPhil Frost

1,9561510




1,9561510












  • +1. If it enables the scenario, a dying planet is perfectly fine. It doesn't matter what happens to the planet after the story ends. :P
    – Devsman
    2 days ago


















  • +1. If it enables the scenario, a dying planet is perfectly fine. It doesn't matter what happens to the planet after the story ends. :P
    – Devsman
    2 days ago
















+1. If it enables the scenario, a dying planet is perfectly fine. It doesn't matter what happens to the planet after the story ends. :P
– Devsman
2 days ago




+1. If it enables the scenario, a dying planet is perfectly fine. It doesn't matter what happens to the planet after the story ends. :P
– Devsman
2 days ago











4














Not entirely sure about the feasibility of this, but it's an idea I had when I read your question. I wonder if something like this would be possible through Gravitational Lensing. Essentially, this is when black holes (with enormous gravitational pulls) bend light around them, causing telescopic effects. I've linked the Wikipedia page for gravitational lensing as well as an article from Space.com below which you could read up on to give you a better idea of how it all works.



My idea though is that what if, just outside the edge of what can be seen from the planet, there's a system of black holes which pull light in such a manner that it's focussed into a thin beam, which cuts across the galaxy and occasionally burns its way across your planet? This would explain the huge intensity of the light as well as allowing a 'natural' explanation for how it's focussed so tightly.



Having it at such a distance would also mean that the appearance of the beam of light can't be predicted, as the people on the planet don't have the technology to either see that far into space, or understand what they're seeing. Besides, at such a distance that the black holes don't mess with the solar system's structure, the light would take very long to reach the planet. So when the beam lines up, its effects are only seen on the planet later (how much later depends on the distance. Centuries, or even millennia maybe).



As for the colour of the beam, we can assume the source of light is moving away from the black holes and the planet, which would cause Redshift. This would make a white light source look red to the observer. Frequency wise, we could assume that other objects in space (planets, dust clouds etc) often block the light from hitting our planet, but occasionally it slips through the gaps (like when you see the sun for a couple of seconds through a clearing in the clouds before it gets blocked again).



For non-animals to survive, perhaps they've evolved to feed on the huge light intensity and maybe even need it every few days to live? Or (depending on the history of the 'humans' on your planet) maybe everything else is evolved to survive the intense light to an extent, while humans aren't. Perhaps this is similar to how we have to wear clothes - we can't handle Earth's natural climates without external help. Maybe those caught outside of their radiation booths are killed rapidly, while those who stay inside are fine?



There's a lot of ways you can go with this idea and I think the rest is up to you. I've included a few links at the bottom you might be interested in.



Further Reading:



Gravitational lensing:



https://www.space.com/39999-how-gravitational-lenses-work.html



https://en.wikipedia.org/wiki/Gravitational_lens



Red and blue shift:



https://en.wikipedia.org/wiki/Redshift



https://www.space.com/25732-redshift-blueshift.html






share|improve this answer





















  • +1 for redshift
    – Devsman
    2 days ago






  • 8




    Sadly, the conservation of etendue makes this system of black hole lenses unworkable. If you're concentrating the light to a smaller output area, then it must also be distributed across a larger angle as it leaves the lens. XKCD's Randall Munroe did an excellent post on this topic: what-if.xkcd.com/145
    – Dubukay
    2 days ago






  • 1




    @Dubukay wow, never heard of that conservation law before. Thanks for bringing it to my attention! Love learning stuff on here haha
    – user43712
    2 days ago






  • 4




    However, if what we're going for is suspension-of-disbelief, make it a mini black hole in an inner orbit of the same system. Each time that BH passes between your unfortunate planet and the sun, lensing burns a scar right across the planet. If there's enough sunlight to start a fire with a magnifying glass, there's enough to do it through gravitational lensing (note that most of the planet will be nearly dark due to the lensing effect).
    – JBH
    2 days ago






  • 5




    Also, keep in mind that the mini black hole could also be one half of a binary star, in which case the orbits are preserved and the effect is the same, if a bit more complicated to calculate.
    – JBH
    2 days ago
















4














Not entirely sure about the feasibility of this, but it's an idea I had when I read your question. I wonder if something like this would be possible through Gravitational Lensing. Essentially, this is when black holes (with enormous gravitational pulls) bend light around them, causing telescopic effects. I've linked the Wikipedia page for gravitational lensing as well as an article from Space.com below which you could read up on to give you a better idea of how it all works.



My idea though is that what if, just outside the edge of what can be seen from the planet, there's a system of black holes which pull light in such a manner that it's focussed into a thin beam, which cuts across the galaxy and occasionally burns its way across your planet? This would explain the huge intensity of the light as well as allowing a 'natural' explanation for how it's focussed so tightly.



Having it at such a distance would also mean that the appearance of the beam of light can't be predicted, as the people on the planet don't have the technology to either see that far into space, or understand what they're seeing. Besides, at such a distance that the black holes don't mess with the solar system's structure, the light would take very long to reach the planet. So when the beam lines up, its effects are only seen on the planet later (how much later depends on the distance. Centuries, or even millennia maybe).



As for the colour of the beam, we can assume the source of light is moving away from the black holes and the planet, which would cause Redshift. This would make a white light source look red to the observer. Frequency wise, we could assume that other objects in space (planets, dust clouds etc) often block the light from hitting our planet, but occasionally it slips through the gaps (like when you see the sun for a couple of seconds through a clearing in the clouds before it gets blocked again).



For non-animals to survive, perhaps they've evolved to feed on the huge light intensity and maybe even need it every few days to live? Or (depending on the history of the 'humans' on your planet) maybe everything else is evolved to survive the intense light to an extent, while humans aren't. Perhaps this is similar to how we have to wear clothes - we can't handle Earth's natural climates without external help. Maybe those caught outside of their radiation booths are killed rapidly, while those who stay inside are fine?



There's a lot of ways you can go with this idea and I think the rest is up to you. I've included a few links at the bottom you might be interested in.



Further Reading:



Gravitational lensing:



https://www.space.com/39999-how-gravitational-lenses-work.html



https://en.wikipedia.org/wiki/Gravitational_lens



Red and blue shift:



https://en.wikipedia.org/wiki/Redshift



https://www.space.com/25732-redshift-blueshift.html






share|improve this answer





















  • +1 for redshift
    – Devsman
    2 days ago






  • 8




    Sadly, the conservation of etendue makes this system of black hole lenses unworkable. If you're concentrating the light to a smaller output area, then it must also be distributed across a larger angle as it leaves the lens. XKCD's Randall Munroe did an excellent post on this topic: what-if.xkcd.com/145
    – Dubukay
    2 days ago






  • 1




    @Dubukay wow, never heard of that conservation law before. Thanks for bringing it to my attention! Love learning stuff on here haha
    – user43712
    2 days ago






  • 4




    However, if what we're going for is suspension-of-disbelief, make it a mini black hole in an inner orbit of the same system. Each time that BH passes between your unfortunate planet and the sun, lensing burns a scar right across the planet. If there's enough sunlight to start a fire with a magnifying glass, there's enough to do it through gravitational lensing (note that most of the planet will be nearly dark due to the lensing effect).
    – JBH
    2 days ago






  • 5




    Also, keep in mind that the mini black hole could also be one half of a binary star, in which case the orbits are preserved and the effect is the same, if a bit more complicated to calculate.
    – JBH
    2 days ago














4












4








4






Not entirely sure about the feasibility of this, but it's an idea I had when I read your question. I wonder if something like this would be possible through Gravitational Lensing. Essentially, this is when black holes (with enormous gravitational pulls) bend light around them, causing telescopic effects. I've linked the Wikipedia page for gravitational lensing as well as an article from Space.com below which you could read up on to give you a better idea of how it all works.



My idea though is that what if, just outside the edge of what can be seen from the planet, there's a system of black holes which pull light in such a manner that it's focussed into a thin beam, which cuts across the galaxy and occasionally burns its way across your planet? This would explain the huge intensity of the light as well as allowing a 'natural' explanation for how it's focussed so tightly.



Having it at such a distance would also mean that the appearance of the beam of light can't be predicted, as the people on the planet don't have the technology to either see that far into space, or understand what they're seeing. Besides, at such a distance that the black holes don't mess with the solar system's structure, the light would take very long to reach the planet. So when the beam lines up, its effects are only seen on the planet later (how much later depends on the distance. Centuries, or even millennia maybe).



As for the colour of the beam, we can assume the source of light is moving away from the black holes and the planet, which would cause Redshift. This would make a white light source look red to the observer. Frequency wise, we could assume that other objects in space (planets, dust clouds etc) often block the light from hitting our planet, but occasionally it slips through the gaps (like when you see the sun for a couple of seconds through a clearing in the clouds before it gets blocked again).



For non-animals to survive, perhaps they've evolved to feed on the huge light intensity and maybe even need it every few days to live? Or (depending on the history of the 'humans' on your planet) maybe everything else is evolved to survive the intense light to an extent, while humans aren't. Perhaps this is similar to how we have to wear clothes - we can't handle Earth's natural climates without external help. Maybe those caught outside of their radiation booths are killed rapidly, while those who stay inside are fine?



There's a lot of ways you can go with this idea and I think the rest is up to you. I've included a few links at the bottom you might be interested in.



Further Reading:



Gravitational lensing:



https://www.space.com/39999-how-gravitational-lenses-work.html



https://en.wikipedia.org/wiki/Gravitational_lens



Red and blue shift:



https://en.wikipedia.org/wiki/Redshift



https://www.space.com/25732-redshift-blueshift.html






share|improve this answer












Not entirely sure about the feasibility of this, but it's an idea I had when I read your question. I wonder if something like this would be possible through Gravitational Lensing. Essentially, this is when black holes (with enormous gravitational pulls) bend light around them, causing telescopic effects. I've linked the Wikipedia page for gravitational lensing as well as an article from Space.com below which you could read up on to give you a better idea of how it all works.



My idea though is that what if, just outside the edge of what can be seen from the planet, there's a system of black holes which pull light in such a manner that it's focussed into a thin beam, which cuts across the galaxy and occasionally burns its way across your planet? This would explain the huge intensity of the light as well as allowing a 'natural' explanation for how it's focussed so tightly.



Having it at such a distance would also mean that the appearance of the beam of light can't be predicted, as the people on the planet don't have the technology to either see that far into space, or understand what they're seeing. Besides, at such a distance that the black holes don't mess with the solar system's structure, the light would take very long to reach the planet. So when the beam lines up, its effects are only seen on the planet later (how much later depends on the distance. Centuries, or even millennia maybe).



As for the colour of the beam, we can assume the source of light is moving away from the black holes and the planet, which would cause Redshift. This would make a white light source look red to the observer. Frequency wise, we could assume that other objects in space (planets, dust clouds etc) often block the light from hitting our planet, but occasionally it slips through the gaps (like when you see the sun for a couple of seconds through a clearing in the clouds before it gets blocked again).



For non-animals to survive, perhaps they've evolved to feed on the huge light intensity and maybe even need it every few days to live? Or (depending on the history of the 'humans' on your planet) maybe everything else is evolved to survive the intense light to an extent, while humans aren't. Perhaps this is similar to how we have to wear clothes - we can't handle Earth's natural climates without external help. Maybe those caught outside of their radiation booths are killed rapidly, while those who stay inside are fine?



There's a lot of ways you can go with this idea and I think the rest is up to you. I've included a few links at the bottom you might be interested in.



Further Reading:



Gravitational lensing:



https://www.space.com/39999-how-gravitational-lenses-work.html



https://en.wikipedia.org/wiki/Gravitational_lens



Red and blue shift:



https://en.wikipedia.org/wiki/Redshift



https://www.space.com/25732-redshift-blueshift.html







share|improve this answer












share|improve this answer



share|improve this answer










answered 2 days ago









user43712user43712

1192




1192












  • +1 for redshift
    – Devsman
    2 days ago






  • 8




    Sadly, the conservation of etendue makes this system of black hole lenses unworkable. If you're concentrating the light to a smaller output area, then it must also be distributed across a larger angle as it leaves the lens. XKCD's Randall Munroe did an excellent post on this topic: what-if.xkcd.com/145
    – Dubukay
    2 days ago






  • 1




    @Dubukay wow, never heard of that conservation law before. Thanks for bringing it to my attention! Love learning stuff on here haha
    – user43712
    2 days ago






  • 4




    However, if what we're going for is suspension-of-disbelief, make it a mini black hole in an inner orbit of the same system. Each time that BH passes between your unfortunate planet and the sun, lensing burns a scar right across the planet. If there's enough sunlight to start a fire with a magnifying glass, there's enough to do it through gravitational lensing (note that most of the planet will be nearly dark due to the lensing effect).
    – JBH
    2 days ago






  • 5




    Also, keep in mind that the mini black hole could also be one half of a binary star, in which case the orbits are preserved and the effect is the same, if a bit more complicated to calculate.
    – JBH
    2 days ago


















  • +1 for redshift
    – Devsman
    2 days ago






  • 8




    Sadly, the conservation of etendue makes this system of black hole lenses unworkable. If you're concentrating the light to a smaller output area, then it must also be distributed across a larger angle as it leaves the lens. XKCD's Randall Munroe did an excellent post on this topic: what-if.xkcd.com/145
    – Dubukay
    2 days ago






  • 1




    @Dubukay wow, never heard of that conservation law before. Thanks for bringing it to my attention! Love learning stuff on here haha
    – user43712
    2 days ago






  • 4




    However, if what we're going for is suspension-of-disbelief, make it a mini black hole in an inner orbit of the same system. Each time that BH passes between your unfortunate planet and the sun, lensing burns a scar right across the planet. If there's enough sunlight to start a fire with a magnifying glass, there's enough to do it through gravitational lensing (note that most of the planet will be nearly dark due to the lensing effect).
    – JBH
    2 days ago






  • 5




    Also, keep in mind that the mini black hole could also be one half of a binary star, in which case the orbits are preserved and the effect is the same, if a bit more complicated to calculate.
    – JBH
    2 days ago
















+1 for redshift
– Devsman
2 days ago




+1 for redshift
– Devsman
2 days ago




8




8




Sadly, the conservation of etendue makes this system of black hole lenses unworkable. If you're concentrating the light to a smaller output area, then it must also be distributed across a larger angle as it leaves the lens. XKCD's Randall Munroe did an excellent post on this topic: what-if.xkcd.com/145
– Dubukay
2 days ago




Sadly, the conservation of etendue makes this system of black hole lenses unworkable. If you're concentrating the light to a smaller output area, then it must also be distributed across a larger angle as it leaves the lens. XKCD's Randall Munroe did an excellent post on this topic: what-if.xkcd.com/145
– Dubukay
2 days ago




1




1




@Dubukay wow, never heard of that conservation law before. Thanks for bringing it to my attention! Love learning stuff on here haha
– user43712
2 days ago




@Dubukay wow, never heard of that conservation law before. Thanks for bringing it to my attention! Love learning stuff on here haha
– user43712
2 days ago




4




4




However, if what we're going for is suspension-of-disbelief, make it a mini black hole in an inner orbit of the same system. Each time that BH passes between your unfortunate planet and the sun, lensing burns a scar right across the planet. If there's enough sunlight to start a fire with a magnifying glass, there's enough to do it through gravitational lensing (note that most of the planet will be nearly dark due to the lensing effect).
– JBH
2 days ago




However, if what we're going for is suspension-of-disbelief, make it a mini black hole in an inner orbit of the same system. Each time that BH passes between your unfortunate planet and the sun, lensing burns a scar right across the planet. If there's enough sunlight to start a fire with a magnifying glass, there's enough to do it through gravitational lensing (note that most of the planet will be nearly dark due to the lensing effect).
– JBH
2 days ago




5




5




Also, keep in mind that the mini black hole could also be one half of a binary star, in which case the orbits are preserved and the effect is the same, if a bit more complicated to calculate.
– JBH
2 days ago




Also, keep in mind that the mini black hole could also be one half of a binary star, in which case the orbits are preserved and the effect is the same, if a bit more complicated to calculate.
– JBH
2 days ago











2














Ozone holes (https://en.wikipedia.org/wiki/Ozone_depletion) already cause real world health problems.



I think you could combine ozone holes and the loss of other atmospheric protection with the a coinciding local weakening of the magnetosphere. In the presence of a very energetic "sun", the results might be "deadly".



"Deadly" as in it might be immediately painful and certainly cancerous over time. Not deadly in the Vin Diesel kind of way you might be looking for though.






share|improve this answer





















  • A magnetosphere has no effect on electromagnetic radiation, and a breathable atmosphere would protect against solar wind.
    – Christopher James Huff
    2 days ago
















2














Ozone holes (https://en.wikipedia.org/wiki/Ozone_depletion) already cause real world health problems.



I think you could combine ozone holes and the loss of other atmospheric protection with the a coinciding local weakening of the magnetosphere. In the presence of a very energetic "sun", the results might be "deadly".



"Deadly" as in it might be immediately painful and certainly cancerous over time. Not deadly in the Vin Diesel kind of way you might be looking for though.






share|improve this answer





















  • A magnetosphere has no effect on electromagnetic radiation, and a breathable atmosphere would protect against solar wind.
    – Christopher James Huff
    2 days ago














2












2








2






Ozone holes (https://en.wikipedia.org/wiki/Ozone_depletion) already cause real world health problems.



I think you could combine ozone holes and the loss of other atmospheric protection with the a coinciding local weakening of the magnetosphere. In the presence of a very energetic "sun", the results might be "deadly".



"Deadly" as in it might be immediately painful and certainly cancerous over time. Not deadly in the Vin Diesel kind of way you might be looking for though.






share|improve this answer












Ozone holes (https://en.wikipedia.org/wiki/Ozone_depletion) already cause real world health problems.



I think you could combine ozone holes and the loss of other atmospheric protection with the a coinciding local weakening of the magnetosphere. In the presence of a very energetic "sun", the results might be "deadly".



"Deadly" as in it might be immediately painful and certainly cancerous over time. Not deadly in the Vin Diesel kind of way you might be looking for though.







share|improve this answer












share|improve this answer



share|improve this answer










answered 2 days ago









JonSGJonSG

45125




45125












  • A magnetosphere has no effect on electromagnetic radiation, and a breathable atmosphere would protect against solar wind.
    – Christopher James Huff
    2 days ago


















  • A magnetosphere has no effect on electromagnetic radiation, and a breathable atmosphere would protect against solar wind.
    – Christopher James Huff
    2 days ago
















A magnetosphere has no effect on electromagnetic radiation, and a breathable atmosphere would protect against solar wind.
– Christopher James Huff
2 days ago




A magnetosphere has no effect on electromagnetic radiation, and a breathable atmosphere would protect against solar wind.
– Christopher James Huff
2 days ago











2














Let's have some cosmic fun.



Say that your planet orbits inside of an expanding red giant star.



Yep.



Now, say that someone or something decided to construct a shell around your world, perhaps before it had been engulfed by your host star as it evolved from the main sequence. This shell may be layers upon layers of orbital rings.



Now, say that this shell--made of some impossible material, probably--has some degree of translucency to it and the ability to change this translucency. You can have one hemisphere of the shell totally opaque to simulate night, with perhaps little points of translucency to simulate stars, and the other hemisphere translucent to simulate daylight (much less a simulation this point, because actual starlight would be entering).



Finally, let this shell have some circular region of total transparency which sweeps the planet, perhaps moving across the shell faster than the day-night cycle to give the folks on all sides, day or night, some of the action. You can think of the shell as being comprised of a bunch of little windows if you'd like, like pixels on a screen, that can be tinted and whatnot and can be made to filter out the unnecessary or harmful radiation of the star it is orbiting inside. The transparent portion would not filter these things, allowing death to beam down onto everything below. Heck, perhaps it even has little perforations or 'openings of the windows' that allow actual, high-energy particles to enter as well.






share|improve this answer



















  • 1




    "Say that your planet orbits inside of an expanding red giant star." It would quickly deorbit due to friction.
    – Renan
    2 days ago






  • 1




    @Renan Not as quickly as you'd think.
    – B.fox
    2 days ago










  • @B.fox, but still very quickly on a scale of planets. A very rough approximation, pretending that the classic drag equation is valid at all speeds, is that an Earth-sized planet will come to a complete stop after about 400 years.
    – Mark
    2 days ago












  • @Mark I'll have to follow up on that. I'm pretty sure the range was somewhere in the hundreds of thousands of years. The outer atmosphere of a red giant star is pretty diffuse, almost insubstantial.
    – B.fox
    yesterday












  • I'm not sure why this solution should depend on the planet being inside the sun. Take our own system, build such a shell around Mercury.
    – Mr Lister
    yesterday


















2














Let's have some cosmic fun.



Say that your planet orbits inside of an expanding red giant star.



Yep.



Now, say that someone or something decided to construct a shell around your world, perhaps before it had been engulfed by your host star as it evolved from the main sequence. This shell may be layers upon layers of orbital rings.



Now, say that this shell--made of some impossible material, probably--has some degree of translucency to it and the ability to change this translucency. You can have one hemisphere of the shell totally opaque to simulate night, with perhaps little points of translucency to simulate stars, and the other hemisphere translucent to simulate daylight (much less a simulation this point, because actual starlight would be entering).



Finally, let this shell have some circular region of total transparency which sweeps the planet, perhaps moving across the shell faster than the day-night cycle to give the folks on all sides, day or night, some of the action. You can think of the shell as being comprised of a bunch of little windows if you'd like, like pixels on a screen, that can be tinted and whatnot and can be made to filter out the unnecessary or harmful radiation of the star it is orbiting inside. The transparent portion would not filter these things, allowing death to beam down onto everything below. Heck, perhaps it even has little perforations or 'openings of the windows' that allow actual, high-energy particles to enter as well.






share|improve this answer



















  • 1




    "Say that your planet orbits inside of an expanding red giant star." It would quickly deorbit due to friction.
    – Renan
    2 days ago






  • 1




    @Renan Not as quickly as you'd think.
    – B.fox
    2 days ago










  • @B.fox, but still very quickly on a scale of planets. A very rough approximation, pretending that the classic drag equation is valid at all speeds, is that an Earth-sized planet will come to a complete stop after about 400 years.
    – Mark
    2 days ago












  • @Mark I'll have to follow up on that. I'm pretty sure the range was somewhere in the hundreds of thousands of years. The outer atmosphere of a red giant star is pretty diffuse, almost insubstantial.
    – B.fox
    yesterday












  • I'm not sure why this solution should depend on the planet being inside the sun. Take our own system, build such a shell around Mercury.
    – Mr Lister
    yesterday
















2












2








2






Let's have some cosmic fun.



Say that your planet orbits inside of an expanding red giant star.



Yep.



Now, say that someone or something decided to construct a shell around your world, perhaps before it had been engulfed by your host star as it evolved from the main sequence. This shell may be layers upon layers of orbital rings.



Now, say that this shell--made of some impossible material, probably--has some degree of translucency to it and the ability to change this translucency. You can have one hemisphere of the shell totally opaque to simulate night, with perhaps little points of translucency to simulate stars, and the other hemisphere translucent to simulate daylight (much less a simulation this point, because actual starlight would be entering).



Finally, let this shell have some circular region of total transparency which sweeps the planet, perhaps moving across the shell faster than the day-night cycle to give the folks on all sides, day or night, some of the action. You can think of the shell as being comprised of a bunch of little windows if you'd like, like pixels on a screen, that can be tinted and whatnot and can be made to filter out the unnecessary or harmful radiation of the star it is orbiting inside. The transparent portion would not filter these things, allowing death to beam down onto everything below. Heck, perhaps it even has little perforations or 'openings of the windows' that allow actual, high-energy particles to enter as well.






share|improve this answer














Let's have some cosmic fun.



Say that your planet orbits inside of an expanding red giant star.



Yep.



Now, say that someone or something decided to construct a shell around your world, perhaps before it had been engulfed by your host star as it evolved from the main sequence. This shell may be layers upon layers of orbital rings.



Now, say that this shell--made of some impossible material, probably--has some degree of translucency to it and the ability to change this translucency. You can have one hemisphere of the shell totally opaque to simulate night, with perhaps little points of translucency to simulate stars, and the other hemisphere translucent to simulate daylight (much less a simulation this point, because actual starlight would be entering).



Finally, let this shell have some circular region of total transparency which sweeps the planet, perhaps moving across the shell faster than the day-night cycle to give the folks on all sides, day or night, some of the action. You can think of the shell as being comprised of a bunch of little windows if you'd like, like pixels on a screen, that can be tinted and whatnot and can be made to filter out the unnecessary or harmful radiation of the star it is orbiting inside. The transparent portion would not filter these things, allowing death to beam down onto everything below. Heck, perhaps it even has little perforations or 'openings of the windows' that allow actual, high-energy particles to enter as well.







share|improve this answer














share|improve this answer



share|improve this answer








edited 2 days ago

























answered 2 days ago









B.foxB.fox

9491316




9491316








  • 1




    "Say that your planet orbits inside of an expanding red giant star." It would quickly deorbit due to friction.
    – Renan
    2 days ago






  • 1




    @Renan Not as quickly as you'd think.
    – B.fox
    2 days ago










  • @B.fox, but still very quickly on a scale of planets. A very rough approximation, pretending that the classic drag equation is valid at all speeds, is that an Earth-sized planet will come to a complete stop after about 400 years.
    – Mark
    2 days ago












  • @Mark I'll have to follow up on that. I'm pretty sure the range was somewhere in the hundreds of thousands of years. The outer atmosphere of a red giant star is pretty diffuse, almost insubstantial.
    – B.fox
    yesterday












  • I'm not sure why this solution should depend on the planet being inside the sun. Take our own system, build such a shell around Mercury.
    – Mr Lister
    yesterday
















  • 1




    "Say that your planet orbits inside of an expanding red giant star." It would quickly deorbit due to friction.
    – Renan
    2 days ago






  • 1




    @Renan Not as quickly as you'd think.
    – B.fox
    2 days ago










  • @B.fox, but still very quickly on a scale of planets. A very rough approximation, pretending that the classic drag equation is valid at all speeds, is that an Earth-sized planet will come to a complete stop after about 400 years.
    – Mark
    2 days ago












  • @Mark I'll have to follow up on that. I'm pretty sure the range was somewhere in the hundreds of thousands of years. The outer atmosphere of a red giant star is pretty diffuse, almost insubstantial.
    – B.fox
    yesterday












  • I'm not sure why this solution should depend on the planet being inside the sun. Take our own system, build such a shell around Mercury.
    – Mr Lister
    yesterday










1




1




"Say that your planet orbits inside of an expanding red giant star." It would quickly deorbit due to friction.
– Renan
2 days ago




"Say that your planet orbits inside of an expanding red giant star." It would quickly deorbit due to friction.
– Renan
2 days ago




1




1




@Renan Not as quickly as you'd think.
– B.fox
2 days ago




@Renan Not as quickly as you'd think.
– B.fox
2 days ago












@B.fox, but still very quickly on a scale of planets. A very rough approximation, pretending that the classic drag equation is valid at all speeds, is that an Earth-sized planet will come to a complete stop after about 400 years.
– Mark
2 days ago






@B.fox, but still very quickly on a scale of planets. A very rough approximation, pretending that the classic drag equation is valid at all speeds, is that an Earth-sized planet will come to a complete stop after about 400 years.
– Mark
2 days ago














@Mark I'll have to follow up on that. I'm pretty sure the range was somewhere in the hundreds of thousands of years. The outer atmosphere of a red giant star is pretty diffuse, almost insubstantial.
– B.fox
yesterday






@Mark I'll have to follow up on that. I'm pretty sure the range was somewhere in the hundreds of thousands of years. The outer atmosphere of a red giant star is pretty diffuse, almost insubstantial.
– B.fox
yesterday














I'm not sure why this solution should depend on the planet being inside the sun. Take our own system, build such a shell around Mercury.
– Mr Lister
yesterday






I'm not sure why this solution should depend on the planet being inside the sun. Take our own system, build such a shell around Mercury.
– Mr Lister
yesterday













1














Constant lightning.



Consider a Rocheworld.
Can an atmosphere englobe a planetary ring?




Two tidally locked planets just outside the Roche Limit can orbit each
other and share a combined atmosphere. You would be able to fly from
one to the other without ever leaving the atmosphere and objects
placed at the lagrange points would be able to remain there.




These binary planets circle around each other. At one point in their orbit, their atmospheres touch (or you could have a moon graze the atmosphere of its planet). The friction of the two atmospheres against each other produces colossal amounts of electrical charge. When the two are close enough that the atmosphere can provide a path, charge can equilibrate across.



This takes the form of constant tremendous bolts of lightning that follow the path of the point where the two partners are closest to one another.






share|improve this answer





















  • I've never heard of a Rocheworld before... thats pretty cool!
    – Corbin Matheson
    17 hours ago
















1














Constant lightning.



Consider a Rocheworld.
Can an atmosphere englobe a planetary ring?




Two tidally locked planets just outside the Roche Limit can orbit each
other and share a combined atmosphere. You would be able to fly from
one to the other without ever leaving the atmosphere and objects
placed at the lagrange points would be able to remain there.




These binary planets circle around each other. At one point in their orbit, their atmospheres touch (or you could have a moon graze the atmosphere of its planet). The friction of the two atmospheres against each other produces colossal amounts of electrical charge. When the two are close enough that the atmosphere can provide a path, charge can equilibrate across.



This takes the form of constant tremendous bolts of lightning that follow the path of the point where the two partners are closest to one another.






share|improve this answer





















  • I've never heard of a Rocheworld before... thats pretty cool!
    – Corbin Matheson
    17 hours ago














1












1








1






Constant lightning.



Consider a Rocheworld.
Can an atmosphere englobe a planetary ring?




Two tidally locked planets just outside the Roche Limit can orbit each
other and share a combined atmosphere. You would be able to fly from
one to the other without ever leaving the atmosphere and objects
placed at the lagrange points would be able to remain there.




These binary planets circle around each other. At one point in their orbit, their atmospheres touch (or you could have a moon graze the atmosphere of its planet). The friction of the two atmospheres against each other produces colossal amounts of electrical charge. When the two are close enough that the atmosphere can provide a path, charge can equilibrate across.



This takes the form of constant tremendous bolts of lightning that follow the path of the point where the two partners are closest to one another.






share|improve this answer












Constant lightning.



Consider a Rocheworld.
Can an atmosphere englobe a planetary ring?




Two tidally locked planets just outside the Roche Limit can orbit each
other and share a combined atmosphere. You would be able to fly from
one to the other without ever leaving the atmosphere and objects
placed at the lagrange points would be able to remain there.




These binary planets circle around each other. At one point in their orbit, their atmospheres touch (or you could have a moon graze the atmosphere of its planet). The friction of the two atmospheres against each other produces colossal amounts of electrical charge. When the two are close enough that the atmosphere can provide a path, charge can equilibrate across.



This takes the form of constant tremendous bolts of lightning that follow the path of the point where the two partners are closest to one another.







share|improve this answer












share|improve this answer



share|improve this answer










answered 2 days ago









WillkWillk

103k25197433




103k25197433












  • I've never heard of a Rocheworld before... thats pretty cool!
    – Corbin Matheson
    17 hours ago


















  • I've never heard of a Rocheworld before... thats pretty cool!
    – Corbin Matheson
    17 hours ago
















I've never heard of a Rocheworld before... thats pretty cool!
– Corbin Matheson
17 hours ago




I've never heard of a Rocheworld before... thats pretty cool!
– Corbin Matheson
17 hours ago











1














I haven't seen this directly addressed, so I'll pose it as an answer:



TL;DR: Large moon with atmosphere refracts to a "point" on your planet. See below for etendue/thermodynamics, refraction, periodicity, and "warning signs".



A companion (moon, twin planet, or even planet as primary with your story set on a comfortable moon of a gas giant) large enough to hold a substantial atmosphere can perhaps be tuned to get the result you need.



Devise an atmosphere for the companion body with a powerful thermal inversion somewhere that reduces some of the spreading due to typical refraction of a density-stratified lens.



So we effectively have a ring-shaped lens, fairly narrow (edge view of the companion's atmosphere) but of very wide diameter (the companion itself), tuned to refract fairly well to a "point". The source of the light is the sun, and we will not get hotter than that. We do not need perfect point focus, but will gladly accept a central line of foci for various degrees of refraction, which generate -- you guessed it -- different colors of spotlight at different orbital distance of the large body from your planet's surface. Blue when it's close, red when it's far -- if it behaves like a proper lens-shaped lens. This would also result in color change as the effect sweeps from the edge of the home planet (farther) to the center (a bit closer). Warning signs would be similar to normal eclipses (the effect would only be observable from the very height of the eclipse). Finally, a combination of rotational and orbital planes for the three bodies involved can do wonders for making a simple periodic set of processes appear miserably non-periodic, particularly for observers located at different points on the surface of the home body.



I'll create a graphic. But I think this thing is doable with a lot less machinery than has been proposed so far, and without violating physics to the point of ridicule.






share|improve this answer



















  • 1




    The only reason this won't get ridiculed for violating physics is that most people don't know about conservation of etendue.
    – Mark
    2 days ago










  • Mark, how so? A garden variety magnifying glass does not get as hot as the point focused upon.
    – Haakon Dahl
    21 hours ago










  • Very, very simplified explanation of why "conservation of etendue" keeps this from working: the Sun is a small, very hot patch of sky surrounded by a whole lot of not-hot sky, so the ground doesn't normally get very hot. A magnifying glass heats things by creating a "virtual Sun" the size of the magnifying glass and exactly as hot as the Sun. This "virtual Sun" takes up a much larger part of the target's view of the sky, so the target gets hotter. Your lens planet, though, takes up only a small part of the view of the sky, and so doesn't do much more heating than the Sun, if that.
    – Mark
    9 hours ago
















1














I haven't seen this directly addressed, so I'll pose it as an answer:



TL;DR: Large moon with atmosphere refracts to a "point" on your planet. See below for etendue/thermodynamics, refraction, periodicity, and "warning signs".



A companion (moon, twin planet, or even planet as primary with your story set on a comfortable moon of a gas giant) large enough to hold a substantial atmosphere can perhaps be tuned to get the result you need.



Devise an atmosphere for the companion body with a powerful thermal inversion somewhere that reduces some of the spreading due to typical refraction of a density-stratified lens.



So we effectively have a ring-shaped lens, fairly narrow (edge view of the companion's atmosphere) but of very wide diameter (the companion itself), tuned to refract fairly well to a "point". The source of the light is the sun, and we will not get hotter than that. We do not need perfect point focus, but will gladly accept a central line of foci for various degrees of refraction, which generate -- you guessed it -- different colors of spotlight at different orbital distance of the large body from your planet's surface. Blue when it's close, red when it's far -- if it behaves like a proper lens-shaped lens. This would also result in color change as the effect sweeps from the edge of the home planet (farther) to the center (a bit closer). Warning signs would be similar to normal eclipses (the effect would only be observable from the very height of the eclipse). Finally, a combination of rotational and orbital planes for the three bodies involved can do wonders for making a simple periodic set of processes appear miserably non-periodic, particularly for observers located at different points on the surface of the home body.



I'll create a graphic. But I think this thing is doable with a lot less machinery than has been proposed so far, and without violating physics to the point of ridicule.






share|improve this answer



















  • 1




    The only reason this won't get ridiculed for violating physics is that most people don't know about conservation of etendue.
    – Mark
    2 days ago










  • Mark, how so? A garden variety magnifying glass does not get as hot as the point focused upon.
    – Haakon Dahl
    21 hours ago










  • Very, very simplified explanation of why "conservation of etendue" keeps this from working: the Sun is a small, very hot patch of sky surrounded by a whole lot of not-hot sky, so the ground doesn't normally get very hot. A magnifying glass heats things by creating a "virtual Sun" the size of the magnifying glass and exactly as hot as the Sun. This "virtual Sun" takes up a much larger part of the target's view of the sky, so the target gets hotter. Your lens planet, though, takes up only a small part of the view of the sky, and so doesn't do much more heating than the Sun, if that.
    – Mark
    9 hours ago














1












1








1






I haven't seen this directly addressed, so I'll pose it as an answer:



TL;DR: Large moon with atmosphere refracts to a "point" on your planet. See below for etendue/thermodynamics, refraction, periodicity, and "warning signs".



A companion (moon, twin planet, or even planet as primary with your story set on a comfortable moon of a gas giant) large enough to hold a substantial atmosphere can perhaps be tuned to get the result you need.



Devise an atmosphere for the companion body with a powerful thermal inversion somewhere that reduces some of the spreading due to typical refraction of a density-stratified lens.



So we effectively have a ring-shaped lens, fairly narrow (edge view of the companion's atmosphere) but of very wide diameter (the companion itself), tuned to refract fairly well to a "point". The source of the light is the sun, and we will not get hotter than that. We do not need perfect point focus, but will gladly accept a central line of foci for various degrees of refraction, which generate -- you guessed it -- different colors of spotlight at different orbital distance of the large body from your planet's surface. Blue when it's close, red when it's far -- if it behaves like a proper lens-shaped lens. This would also result in color change as the effect sweeps from the edge of the home planet (farther) to the center (a bit closer). Warning signs would be similar to normal eclipses (the effect would only be observable from the very height of the eclipse). Finally, a combination of rotational and orbital planes for the three bodies involved can do wonders for making a simple periodic set of processes appear miserably non-periodic, particularly for observers located at different points on the surface of the home body.



I'll create a graphic. But I think this thing is doable with a lot less machinery than has been proposed so far, and without violating physics to the point of ridicule.






share|improve this answer














I haven't seen this directly addressed, so I'll pose it as an answer:



TL;DR: Large moon with atmosphere refracts to a "point" on your planet. See below for etendue/thermodynamics, refraction, periodicity, and "warning signs".



A companion (moon, twin planet, or even planet as primary with your story set on a comfortable moon of a gas giant) large enough to hold a substantial atmosphere can perhaps be tuned to get the result you need.



Devise an atmosphere for the companion body with a powerful thermal inversion somewhere that reduces some of the spreading due to typical refraction of a density-stratified lens.



So we effectively have a ring-shaped lens, fairly narrow (edge view of the companion's atmosphere) but of very wide diameter (the companion itself), tuned to refract fairly well to a "point". The source of the light is the sun, and we will not get hotter than that. We do not need perfect point focus, but will gladly accept a central line of foci for various degrees of refraction, which generate -- you guessed it -- different colors of spotlight at different orbital distance of the large body from your planet's surface. Blue when it's close, red when it's far -- if it behaves like a proper lens-shaped lens. This would also result in color change as the effect sweeps from the edge of the home planet (farther) to the center (a bit closer). Warning signs would be similar to normal eclipses (the effect would only be observable from the very height of the eclipse). Finally, a combination of rotational and orbital planes for the three bodies involved can do wonders for making a simple periodic set of processes appear miserably non-periodic, particularly for observers located at different points on the surface of the home body.



I'll create a graphic. But I think this thing is doable with a lot less machinery than has been proposed so far, and without violating physics to the point of ridicule.







share|improve this answer














share|improve this answer



share|improve this answer








edited 2 days ago

























answered 2 days ago









Haakon DahlHaakon Dahl

31315




31315








  • 1




    The only reason this won't get ridiculed for violating physics is that most people don't know about conservation of etendue.
    – Mark
    2 days ago










  • Mark, how so? A garden variety magnifying glass does not get as hot as the point focused upon.
    – Haakon Dahl
    21 hours ago










  • Very, very simplified explanation of why "conservation of etendue" keeps this from working: the Sun is a small, very hot patch of sky surrounded by a whole lot of not-hot sky, so the ground doesn't normally get very hot. A magnifying glass heats things by creating a "virtual Sun" the size of the magnifying glass and exactly as hot as the Sun. This "virtual Sun" takes up a much larger part of the target's view of the sky, so the target gets hotter. Your lens planet, though, takes up only a small part of the view of the sky, and so doesn't do much more heating than the Sun, if that.
    – Mark
    9 hours ago














  • 1




    The only reason this won't get ridiculed for violating physics is that most people don't know about conservation of etendue.
    – Mark
    2 days ago










  • Mark, how so? A garden variety magnifying glass does not get as hot as the point focused upon.
    – Haakon Dahl
    21 hours ago










  • Very, very simplified explanation of why "conservation of etendue" keeps this from working: the Sun is a small, very hot patch of sky surrounded by a whole lot of not-hot sky, so the ground doesn't normally get very hot. A magnifying glass heats things by creating a "virtual Sun" the size of the magnifying glass and exactly as hot as the Sun. This "virtual Sun" takes up a much larger part of the target's view of the sky, so the target gets hotter. Your lens planet, though, takes up only a small part of the view of the sky, and so doesn't do much more heating than the Sun, if that.
    – Mark
    9 hours ago








1




1




The only reason this won't get ridiculed for violating physics is that most people don't know about conservation of etendue.
– Mark
2 days ago




The only reason this won't get ridiculed for violating physics is that most people don't know about conservation of etendue.
– Mark
2 days ago












Mark, how so? A garden variety magnifying glass does not get as hot as the point focused upon.
– Haakon Dahl
21 hours ago




Mark, how so? A garden variety magnifying glass does not get as hot as the point focused upon.
– Haakon Dahl
21 hours ago












Very, very simplified explanation of why "conservation of etendue" keeps this from working: the Sun is a small, very hot patch of sky surrounded by a whole lot of not-hot sky, so the ground doesn't normally get very hot. A magnifying glass heats things by creating a "virtual Sun" the size of the magnifying glass and exactly as hot as the Sun. This "virtual Sun" takes up a much larger part of the target's view of the sky, so the target gets hotter. Your lens planet, though, takes up only a small part of the view of the sky, and so doesn't do much more heating than the Sun, if that.
– Mark
9 hours ago




Very, very simplified explanation of why "conservation of etendue" keeps this from working: the Sun is a small, very hot patch of sky surrounded by a whole lot of not-hot sky, so the ground doesn't normally get very hot. A magnifying glass heats things by creating a "virtual Sun" the size of the magnifying glass and exactly as hot as the Sun. This "virtual Sun" takes up a much larger part of the target's view of the sky, so the target gets hotter. Your lens planet, though, takes up only a small part of the view of the sky, and so doesn't do much more heating than the Sun, if that.
– Mark
9 hours ago











0














The moon discussion referenced by @Phil Frost suggests part of the answer. A moon is too small so the lens-like body or phenomenon has to be big enough to cover all or most of the sky from the point of view of the target planet (which may itself be just a moon in a bigger system).



The problem is coming up with a celestial lens. If you can solve that, the rest is just a question of placing the target planet and the radiation source at a suitable scale and proximity.



A lens spotlight redirects light from a large area outside the "spot" so the first warning of the death ray's proximity would be a significant darkening, similar to a solar eclipse. In the distance you might see reflections from dust or clouds within the cone of concentrated light, so you can see if it's coming closer.






share|improve this answer








New contributor




maxwellsdemon is a new contributor to this site. Take care in asking for clarification, commenting, and answering.
Check out our Code of Conduct.


















  • You would also see the rim of the magnified sun start to appear from one side as the danger area approached, and could tell if it was going to pass you by or go right over you. But focusing light in such a way requires arrangements of matter too specifically contrived for nature to be a believable explanation. Your best bet would seem to be along the lines of neglectful precursors.
    – Christopher James Huff
    2 days ago
















0














The moon discussion referenced by @Phil Frost suggests part of the answer. A moon is too small so the lens-like body or phenomenon has to be big enough to cover all or most of the sky from the point of view of the target planet (which may itself be just a moon in a bigger system).



The problem is coming up with a celestial lens. If you can solve that, the rest is just a question of placing the target planet and the radiation source at a suitable scale and proximity.



A lens spotlight redirects light from a large area outside the "spot" so the first warning of the death ray's proximity would be a significant darkening, similar to a solar eclipse. In the distance you might see reflections from dust or clouds within the cone of concentrated light, so you can see if it's coming closer.






share|improve this answer








New contributor




maxwellsdemon is a new contributor to this site. Take care in asking for clarification, commenting, and answering.
Check out our Code of Conduct.


















  • You would also see the rim of the magnified sun start to appear from one side as the danger area approached, and could tell if it was going to pass you by or go right over you. But focusing light in such a way requires arrangements of matter too specifically contrived for nature to be a believable explanation. Your best bet would seem to be along the lines of neglectful precursors.
    – Christopher James Huff
    2 days ago














0












0








0






The moon discussion referenced by @Phil Frost suggests part of the answer. A moon is too small so the lens-like body or phenomenon has to be big enough to cover all or most of the sky from the point of view of the target planet (which may itself be just a moon in a bigger system).



The problem is coming up with a celestial lens. If you can solve that, the rest is just a question of placing the target planet and the radiation source at a suitable scale and proximity.



A lens spotlight redirects light from a large area outside the "spot" so the first warning of the death ray's proximity would be a significant darkening, similar to a solar eclipse. In the distance you might see reflections from dust or clouds within the cone of concentrated light, so you can see if it's coming closer.






share|improve this answer








New contributor




maxwellsdemon is a new contributor to this site. Take care in asking for clarification, commenting, and answering.
Check out our Code of Conduct.









The moon discussion referenced by @Phil Frost suggests part of the answer. A moon is too small so the lens-like body or phenomenon has to be big enough to cover all or most of the sky from the point of view of the target planet (which may itself be just a moon in a bigger system).



The problem is coming up with a celestial lens. If you can solve that, the rest is just a question of placing the target planet and the radiation source at a suitable scale and proximity.



A lens spotlight redirects light from a large area outside the "spot" so the first warning of the death ray's proximity would be a significant darkening, similar to a solar eclipse. In the distance you might see reflections from dust or clouds within the cone of concentrated light, so you can see if it's coming closer.







share|improve this answer








New contributor




maxwellsdemon is a new contributor to this site. Take care in asking for clarification, commenting, and answering.
Check out our Code of Conduct.









share|improve this answer



share|improve this answer






New contributor




maxwellsdemon is a new contributor to this site. Take care in asking for clarification, commenting, and answering.
Check out our Code of Conduct.









answered 2 days ago









maxwellsdemonmaxwellsdemon

1




1




New contributor




maxwellsdemon is a new contributor to this site. Take care in asking for clarification, commenting, and answering.
Check out our Code of Conduct.





New contributor





maxwellsdemon is a new contributor to this site. Take care in asking for clarification, commenting, and answering.
Check out our Code of Conduct.






maxwellsdemon is a new contributor to this site. Take care in asking for clarification, commenting, and answering.
Check out our Code of Conduct.












  • You would also see the rim of the magnified sun start to appear from one side as the danger area approached, and could tell if it was going to pass you by or go right over you. But focusing light in such a way requires arrangements of matter too specifically contrived for nature to be a believable explanation. Your best bet would seem to be along the lines of neglectful precursors.
    – Christopher James Huff
    2 days ago


















  • You would also see the rim of the magnified sun start to appear from one side as the danger area approached, and could tell if it was going to pass you by or go right over you. But focusing light in such a way requires arrangements of matter too specifically contrived for nature to be a believable explanation. Your best bet would seem to be along the lines of neglectful precursors.
    – Christopher James Huff
    2 days ago
















You would also see the rim of the magnified sun start to appear from one side as the danger area approached, and could tell if it was going to pass you by or go right over you. But focusing light in such a way requires arrangements of matter too specifically contrived for nature to be a believable explanation. Your best bet would seem to be along the lines of neglectful precursors.
– Christopher James Huff
2 days ago




You would also see the rim of the magnified sun start to appear from one side as the danger area approached, and could tell if it was going to pass you by or go right over you. But focusing light in such a way requires arrangements of matter too specifically contrived for nature to be a believable explanation. Your best bet would seem to be along the lines of neglectful precursors.
– Christopher James Huff
2 days ago











0














The system could be a binary system with a neutron star or black hole orbiting close to the main star but in an eccentric orbit that lasts a few days. When it draws close to the primary it pulls off huge masses of coronal gasses and causes massive incredibly intense solar flares. If these incidents happen at the same time as the planet is in the wrong part of the sky then you can expect some serious pyrotechnics to hit the day side of the planet for a few hours.



The black hole/neutron star would most likely have been captured rather than be an original part of the system, explaining the eccentric orbit and any unusual spin needed etc.



It doesn't take a big stretch to somehow say that the gravity and magnetic fields of the neutron star focuses the ejections into beams somehow. So every X hours you get massive beams of solar energy being fired in random directions. You can then get the variance by saying whether those beams hit your planet or not.






share|improve this answer

















  • 1




    I think this is too deadly. The situation you describe is similar to a classic nova, and the 10,000-fold increase in solar output from one of those will quite handily sterilize a planet.
    – Mark
    yesterday
















0














The system could be a binary system with a neutron star or black hole orbiting close to the main star but in an eccentric orbit that lasts a few days. When it draws close to the primary it pulls off huge masses of coronal gasses and causes massive incredibly intense solar flares. If these incidents happen at the same time as the planet is in the wrong part of the sky then you can expect some serious pyrotechnics to hit the day side of the planet for a few hours.



The black hole/neutron star would most likely have been captured rather than be an original part of the system, explaining the eccentric orbit and any unusual spin needed etc.



It doesn't take a big stretch to somehow say that the gravity and magnetic fields of the neutron star focuses the ejections into beams somehow. So every X hours you get massive beams of solar energy being fired in random directions. You can then get the variance by saying whether those beams hit your planet or not.






share|improve this answer

















  • 1




    I think this is too deadly. The situation you describe is similar to a classic nova, and the 10,000-fold increase in solar output from one of those will quite handily sterilize a planet.
    – Mark
    yesterday














0












0








0






The system could be a binary system with a neutron star or black hole orbiting close to the main star but in an eccentric orbit that lasts a few days. When it draws close to the primary it pulls off huge masses of coronal gasses and causes massive incredibly intense solar flares. If these incidents happen at the same time as the planet is in the wrong part of the sky then you can expect some serious pyrotechnics to hit the day side of the planet for a few hours.



The black hole/neutron star would most likely have been captured rather than be an original part of the system, explaining the eccentric orbit and any unusual spin needed etc.



It doesn't take a big stretch to somehow say that the gravity and magnetic fields of the neutron star focuses the ejections into beams somehow. So every X hours you get massive beams of solar energy being fired in random directions. You can then get the variance by saying whether those beams hit your planet or not.






share|improve this answer












The system could be a binary system with a neutron star or black hole orbiting close to the main star but in an eccentric orbit that lasts a few days. When it draws close to the primary it pulls off huge masses of coronal gasses and causes massive incredibly intense solar flares. If these incidents happen at the same time as the planet is in the wrong part of the sky then you can expect some serious pyrotechnics to hit the day side of the planet for a few hours.



The black hole/neutron star would most likely have been captured rather than be an original part of the system, explaining the eccentric orbit and any unusual spin needed etc.



It doesn't take a big stretch to somehow say that the gravity and magnetic fields of the neutron star focuses the ejections into beams somehow. So every X hours you get massive beams of solar energy being fired in random directions. You can then get the variance by saying whether those beams hit your planet or not.







share|improve this answer












share|improve this answer



share|improve this answer










answered yesterday









Tim BTim B

60k23168288




60k23168288








  • 1




    I think this is too deadly. The situation you describe is similar to a classic nova, and the 10,000-fold increase in solar output from one of those will quite handily sterilize a planet.
    – Mark
    yesterday














  • 1




    I think this is too deadly. The situation you describe is similar to a classic nova, and the 10,000-fold increase in solar output from one of those will quite handily sterilize a planet.
    – Mark
    yesterday








1




1




I think this is too deadly. The situation you describe is similar to a classic nova, and the 10,000-fold increase in solar output from one of those will quite handily sterilize a planet.
– Mark
yesterday




I think this is too deadly. The situation you describe is similar to a classic nova, and the 10,000-fold increase in solar output from one of those will quite handily sterilize a planet.
– Mark
yesterday











-1














A small moon-like object in the planet+star's L1 Lagrange point (i.e., the point where the star's and planet's gravity exactly cancel out) would do the trick. Thanks to the wave nature of light, the moon will generate an Arago spot (bright spot) on the surface of your planet. Pick a small and bright star, perhaps a young white dwarf or neutron star.



To have a moving spot, the moon would need to move about a bit. You can think of an orbit around the L1 point. This should not be too hard.



Much more difficult is the fact that the L1 point is an unstable equilibrium point. Objects do not remain in orbit around the L1 point looking only at gravitational forces. Here, some handwaving is necessary. Perhaps the pressure from stellar winds from the central star have some stabilising influence. Perhaps heating of parts of the planets not obscured by the moon will cause massive out-gassing of the oceans into space, providing a stabilising pressure.



Regardless, it's definitely not a predictable situation, which should make it ideal for your story.






share|improve this answer





















  • This doesn't work for two major reasons. First, you won't get an Arago spot because the Sun isn't even remotely like a point source. And second, even if you did get an Arago spot, it would only be as bright (and as dangerous) as direct sunlight.
    – Mark
    9 hours ago
















-1














A small moon-like object in the planet+star's L1 Lagrange point (i.e., the point where the star's and planet's gravity exactly cancel out) would do the trick. Thanks to the wave nature of light, the moon will generate an Arago spot (bright spot) on the surface of your planet. Pick a small and bright star, perhaps a young white dwarf or neutron star.



To have a moving spot, the moon would need to move about a bit. You can think of an orbit around the L1 point. This should not be too hard.



Much more difficult is the fact that the L1 point is an unstable equilibrium point. Objects do not remain in orbit around the L1 point looking only at gravitational forces. Here, some handwaving is necessary. Perhaps the pressure from stellar winds from the central star have some stabilising influence. Perhaps heating of parts of the planets not obscured by the moon will cause massive out-gassing of the oceans into space, providing a stabilising pressure.



Regardless, it's definitely not a predictable situation, which should make it ideal for your story.






share|improve this answer





















  • This doesn't work for two major reasons. First, you won't get an Arago spot because the Sun isn't even remotely like a point source. And second, even if you did get an Arago spot, it would only be as bright (and as dangerous) as direct sunlight.
    – Mark
    9 hours ago














-1












-1








-1






A small moon-like object in the planet+star's L1 Lagrange point (i.e., the point where the star's and planet's gravity exactly cancel out) would do the trick. Thanks to the wave nature of light, the moon will generate an Arago spot (bright spot) on the surface of your planet. Pick a small and bright star, perhaps a young white dwarf or neutron star.



To have a moving spot, the moon would need to move about a bit. You can think of an orbit around the L1 point. This should not be too hard.



Much more difficult is the fact that the L1 point is an unstable equilibrium point. Objects do not remain in orbit around the L1 point looking only at gravitational forces. Here, some handwaving is necessary. Perhaps the pressure from stellar winds from the central star have some stabilising influence. Perhaps heating of parts of the planets not obscured by the moon will cause massive out-gassing of the oceans into space, providing a stabilising pressure.



Regardless, it's definitely not a predictable situation, which should make it ideal for your story.






share|improve this answer












A small moon-like object in the planet+star's L1 Lagrange point (i.e., the point where the star's and planet's gravity exactly cancel out) would do the trick. Thanks to the wave nature of light, the moon will generate an Arago spot (bright spot) on the surface of your planet. Pick a small and bright star, perhaps a young white dwarf or neutron star.



To have a moving spot, the moon would need to move about a bit. You can think of an orbit around the L1 point. This should not be too hard.



Much more difficult is the fact that the L1 point is an unstable equilibrium point. Objects do not remain in orbit around the L1 point looking only at gravitational forces. Here, some handwaving is necessary. Perhaps the pressure from stellar winds from the central star have some stabilising influence. Perhaps heating of parts of the planets not obscured by the moon will cause massive out-gassing of the oceans into space, providing a stabilising pressure.



Regardless, it's definitely not a predictable situation, which should make it ideal for your story.







share|improve this answer












share|improve this answer



share|improve this answer










answered 17 hours ago









SanchisesSanchises

90249




90249












  • This doesn't work for two major reasons. First, you won't get an Arago spot because the Sun isn't even remotely like a point source. And second, even if you did get an Arago spot, it would only be as bright (and as dangerous) as direct sunlight.
    – Mark
    9 hours ago


















  • This doesn't work for two major reasons. First, you won't get an Arago spot because the Sun isn't even remotely like a point source. And second, even if you did get an Arago spot, it would only be as bright (and as dangerous) as direct sunlight.
    – Mark
    9 hours ago
















This doesn't work for two major reasons. First, you won't get an Arago spot because the Sun isn't even remotely like a point source. And second, even if you did get an Arago spot, it would only be as bright (and as dangerous) as direct sunlight.
– Mark
9 hours ago




This doesn't work for two major reasons. First, you won't get an Arago spot because the Sun isn't even remotely like a point source. And second, even if you did get an Arago spot, it would only be as bright (and as dangerous) as direct sunlight.
– Mark
9 hours ago











-2














Reverse Solar Eclipse.

An orbiting Ocean, say a liquid helium moon of the correct size. Meaning, blocking the entire star, or larger. It could create a focusing and penumbra effect based on your need for destruction. Reinforce your physics.






share|improve this answer










New contributor




SiGGER is a new contributor to this site. Take care in asking for clarification, commenting, and answering.
Check out our Code of Conduct.














  • 2




    The correct size is roughly "fills the entire sky". See conservation of etendue, XKCD What-If #145, and the answers to this question.
    – Mark
    2 days ago
















-2














Reverse Solar Eclipse.

An orbiting Ocean, say a liquid helium moon of the correct size. Meaning, blocking the entire star, or larger. It could create a focusing and penumbra effect based on your need for destruction. Reinforce your physics.






share|improve this answer










New contributor




SiGGER is a new contributor to this site. Take care in asking for clarification, commenting, and answering.
Check out our Code of Conduct.














  • 2




    The correct size is roughly "fills the entire sky". See conservation of etendue, XKCD What-If #145, and the answers to this question.
    – Mark
    2 days ago














-2












-2








-2






Reverse Solar Eclipse.

An orbiting Ocean, say a liquid helium moon of the correct size. Meaning, blocking the entire star, or larger. It could create a focusing and penumbra effect based on your need for destruction. Reinforce your physics.






share|improve this answer










New contributor




SiGGER is a new contributor to this site. Take care in asking for clarification, commenting, and answering.
Check out our Code of Conduct.









Reverse Solar Eclipse.

An orbiting Ocean, say a liquid helium moon of the correct size. Meaning, blocking the entire star, or larger. It could create a focusing and penumbra effect based on your need for destruction. Reinforce your physics.







share|improve this answer










New contributor




SiGGER is a new contributor to this site. Take care in asking for clarification, commenting, and answering.
Check out our Code of Conduct.









share|improve this answer



share|improve this answer








edited yesterday





















New contributor




SiGGER is a new contributor to this site. Take care in asking for clarification, commenting, and answering.
Check out our Code of Conduct.









answered 2 days ago









SiGGERSiGGER

232




232




New contributor




SiGGER is a new contributor to this site. Take care in asking for clarification, commenting, and answering.
Check out our Code of Conduct.





New contributor





SiGGER is a new contributor to this site. Take care in asking for clarification, commenting, and answering.
Check out our Code of Conduct.






SiGGER is a new contributor to this site. Take care in asking for clarification, commenting, and answering.
Check out our Code of Conduct.








  • 2




    The correct size is roughly "fills the entire sky". See conservation of etendue, XKCD What-If #145, and the answers to this question.
    – Mark
    2 days ago














  • 2




    The correct size is roughly "fills the entire sky". See conservation of etendue, XKCD What-If #145, and the answers to this question.
    – Mark
    2 days ago








2




2




The correct size is roughly "fills the entire sky". See conservation of etendue, XKCD What-If #145, and the answers to this question.
– Mark
2 days ago




The correct size is roughly "fills the entire sky". See conservation of etendue, XKCD What-If #145, and the answers to this question.
– Mark
2 days ago











-3














A transparent sphere works as a burning glass so a moon of (impossible) clear material should do the trick by concentrating the rays from the sun if it orbited at the right distance.



Trouble is that absorbtion would eat most of the light if the diameter was more than a kilometer. A thin ice-shell might work, but good luck with explaining the origin (and stability!!) of that ;-)






share|improve this answer








New contributor




Mads Horn is a new contributor to this site. Take care in asking for clarification, commenting, and answering.
Check out our Code of Conduct.


















  • Welcome to Worldbuilding, Mads Horn! If you have a moment, please take the tour and visit the help center to learn more about the site. You may also find Worldbuilding Meta and The Sandbox useful. Here is a meta post on the culture and style of Worldbuilding.SE, just to help you understand our scope and methods, and how we do things here. Have fun!
    – Gryphon
    2 days ago






  • 1




    A thin layer of ice may let enough light through, but it won't have any significant lensing effect, not to mention it wouldn't survive without collapsing spectacularly for more than a few days after being conjured at best, let alone form naturally in the first place.
    – John Dvorak
    2 days ago






  • 1




    "If it orbited at the right distance": the focal distance of a ball lens is $f = nD / 4(n - 1)$, with n being the index of refraction of the material and D the diameter of the sphere. For glass, this works out at about 0.8 D, so that orbit must be very close to the surface. Not to mention that the focus lies on the optical axis, so that it wont fall on the surface unless the moon is in conjuction with the Sun. And ball lenses are horrible lenses, they won't focus the light in a nice focal spot.
    – AlexP
    2 days ago








  • 6




    Won't work, for reasons more than just absorption. See Would a Moon made of water pose a threat to Earth during eclipses?
    – Phil Frost
    2 days ago


















-3














A transparent sphere works as a burning glass so a moon of (impossible) clear material should do the trick by concentrating the rays from the sun if it orbited at the right distance.



Trouble is that absorbtion would eat most of the light if the diameter was more than a kilometer. A thin ice-shell might work, but good luck with explaining the origin (and stability!!) of that ;-)






share|improve this answer








New contributor




Mads Horn is a new contributor to this site. Take care in asking for clarification, commenting, and answering.
Check out our Code of Conduct.


















  • Welcome to Worldbuilding, Mads Horn! If you have a moment, please take the tour and visit the help center to learn more about the site. You may also find Worldbuilding Meta and The Sandbox useful. Here is a meta post on the culture and style of Worldbuilding.SE, just to help you understand our scope and methods, and how we do things here. Have fun!
    – Gryphon
    2 days ago






  • 1




    A thin layer of ice may let enough light through, but it won't have any significant lensing effect, not to mention it wouldn't survive without collapsing spectacularly for more than a few days after being conjured at best, let alone form naturally in the first place.
    – John Dvorak
    2 days ago






  • 1




    "If it orbited at the right distance": the focal distance of a ball lens is $f = nD / 4(n - 1)$, with n being the index of refraction of the material and D the diameter of the sphere. For glass, this works out at about 0.8 D, so that orbit must be very close to the surface. Not to mention that the focus lies on the optical axis, so that it wont fall on the surface unless the moon is in conjuction with the Sun. And ball lenses are horrible lenses, they won't focus the light in a nice focal spot.
    – AlexP
    2 days ago








  • 6




    Won't work, for reasons more than just absorption. See Would a Moon made of water pose a threat to Earth during eclipses?
    – Phil Frost
    2 days ago
















-3












-3








-3






A transparent sphere works as a burning glass so a moon of (impossible) clear material should do the trick by concentrating the rays from the sun if it orbited at the right distance.



Trouble is that absorbtion would eat most of the light if the diameter was more than a kilometer. A thin ice-shell might work, but good luck with explaining the origin (and stability!!) of that ;-)






share|improve this answer








New contributor




Mads Horn is a new contributor to this site. Take care in asking for clarification, commenting, and answering.
Check out our Code of Conduct.









A transparent sphere works as a burning glass so a moon of (impossible) clear material should do the trick by concentrating the rays from the sun if it orbited at the right distance.



Trouble is that absorbtion would eat most of the light if the diameter was more than a kilometer. A thin ice-shell might work, but good luck with explaining the origin (and stability!!) of that ;-)







share|improve this answer








New contributor




Mads Horn is a new contributor to this site. Take care in asking for clarification, commenting, and answering.
Check out our Code of Conduct.









share|improve this answer



share|improve this answer






New contributor




Mads Horn is a new contributor to this site. Take care in asking for clarification, commenting, and answering.
Check out our Code of Conduct.









answered 2 days ago









Mads HornMads Horn

31




31




New contributor




Mads Horn is a new contributor to this site. Take care in asking for clarification, commenting, and answering.
Check out our Code of Conduct.





New contributor





Mads Horn is a new contributor to this site. Take care in asking for clarification, commenting, and answering.
Check out our Code of Conduct.






Mads Horn is a new contributor to this site. Take care in asking for clarification, commenting, and answering.
Check out our Code of Conduct.












  • Welcome to Worldbuilding, Mads Horn! If you have a moment, please take the tour and visit the help center to learn more about the site. You may also find Worldbuilding Meta and The Sandbox useful. Here is a meta post on the culture and style of Worldbuilding.SE, just to help you understand our scope and methods, and how we do things here. Have fun!
    – Gryphon
    2 days ago






  • 1




    A thin layer of ice may let enough light through, but it won't have any significant lensing effect, not to mention it wouldn't survive without collapsing spectacularly for more than a few days after being conjured at best, let alone form naturally in the first place.
    – John Dvorak
    2 days ago






  • 1




    "If it orbited at the right distance": the focal distance of a ball lens is $f = nD / 4(n - 1)$, with n being the index of refraction of the material and D the diameter of the sphere. For glass, this works out at about 0.8 D, so that orbit must be very close to the surface. Not to mention that the focus lies on the optical axis, so that it wont fall on the surface unless the moon is in conjuction with the Sun. And ball lenses are horrible lenses, they won't focus the light in a nice focal spot.
    – AlexP
    2 days ago








  • 6




    Won't work, for reasons more than just absorption. See Would a Moon made of water pose a threat to Earth during eclipses?
    – Phil Frost
    2 days ago




















  • Welcome to Worldbuilding, Mads Horn! If you have a moment, please take the tour and visit the help center to learn more about the site. You may also find Worldbuilding Meta and The Sandbox useful. Here is a meta post on the culture and style of Worldbuilding.SE, just to help you understand our scope and methods, and how we do things here. Have fun!
    – Gryphon
    2 days ago






  • 1




    A thin layer of ice may let enough light through, but it won't have any significant lensing effect, not to mention it wouldn't survive without collapsing spectacularly for more than a few days after being conjured at best, let alone form naturally in the first place.
    – John Dvorak
    2 days ago






  • 1




    "If it orbited at the right distance": the focal distance of a ball lens is $f = nD / 4(n - 1)$, with n being the index of refraction of the material and D the diameter of the sphere. For glass, this works out at about 0.8 D, so that orbit must be very close to the surface. Not to mention that the focus lies on the optical axis, so that it wont fall on the surface unless the moon is in conjuction with the Sun. And ball lenses are horrible lenses, they won't focus the light in a nice focal spot.
    – AlexP
    2 days ago








  • 6




    Won't work, for reasons more than just absorption. See Would a Moon made of water pose a threat to Earth during eclipses?
    – Phil Frost
    2 days ago


















Welcome to Worldbuilding, Mads Horn! If you have a moment, please take the tour and visit the help center to learn more about the site. You may also find Worldbuilding Meta and The Sandbox useful. Here is a meta post on the culture and style of Worldbuilding.SE, just to help you understand our scope and methods, and how we do things here. Have fun!
– Gryphon
2 days ago




Welcome to Worldbuilding, Mads Horn! If you have a moment, please take the tour and visit the help center to learn more about the site. You may also find Worldbuilding Meta and The Sandbox useful. Here is a meta post on the culture and style of Worldbuilding.SE, just to help you understand our scope and methods, and how we do things here. Have fun!
– Gryphon
2 days ago




1




1




A thin layer of ice may let enough light through, but it won't have any significant lensing effect, not to mention it wouldn't survive without collapsing spectacularly for more than a few days after being conjured at best, let alone form naturally in the first place.
– John Dvorak
2 days ago




A thin layer of ice may let enough light through, but it won't have any significant lensing effect, not to mention it wouldn't survive without collapsing spectacularly for more than a few days after being conjured at best, let alone form naturally in the first place.
– John Dvorak
2 days ago




1




1




"If it orbited at the right distance": the focal distance of a ball lens is $f = nD / 4(n - 1)$, with n being the index of refraction of the material and D the diameter of the sphere. For glass, this works out at about 0.8 D, so that orbit must be very close to the surface. Not to mention that the focus lies on the optical axis, so that it wont fall on the surface unless the moon is in conjuction with the Sun. And ball lenses are horrible lenses, they won't focus the light in a nice focal spot.
– AlexP
2 days ago






"If it orbited at the right distance": the focal distance of a ball lens is $f = nD / 4(n - 1)$, with n being the index of refraction of the material and D the diameter of the sphere. For glass, this works out at about 0.8 D, so that orbit must be very close to the surface. Not to mention that the focus lies on the optical axis, so that it wont fall on the surface unless the moon is in conjuction with the Sun. And ball lenses are horrible lenses, they won't focus the light in a nice focal spot.
– AlexP
2 days ago






6




6




Won't work, for reasons more than just absorption. See Would a Moon made of water pose a threat to Earth during eclipses?
– Phil Frost
2 days ago






Won't work, for reasons more than just absorption. See Would a Moon made of water pose a threat to Earth during eclipses?
– Phil Frost
2 days ago




















draft saved

draft discarded




















































Thanks for contributing an answer to Worldbuilding Stack Exchange!


  • Please be sure to answer the question. Provide details and share your research!

But avoid



  • Asking for help, clarification, or responding to other answers.

  • Making statements based on opinion; back them up with references or personal experience.


Use MathJax to format equations. MathJax reference.


To learn more, see our tips on writing great answers.





Some of your past answers have not been well-received, and you're in danger of being blocked from answering.


Please pay close attention to the following guidance:


  • Please be sure to answer the question. Provide details and share your research!

But avoid



  • Asking for help, clarification, or responding to other answers.

  • Making statements based on opinion; back them up with references or personal experience.


To learn more, see our tips on writing great answers.




draft saved


draft discarded














StackExchange.ready(
function () {
StackExchange.openid.initPostLogin('.new-post-login', 'https%3a%2f%2fworldbuilding.stackexchange.com%2fquestions%2f135840%2fhow-can-a-planet-have-a-deadly-eclipse-like-spotlight%23new-answer', 'question_page');
}
);

Post as a guest















Required, but never shown





















































Required, but never shown














Required, but never shown












Required, but never shown







Required, but never shown

































Required, but never shown














Required, but never shown












Required, but never shown







Required, but never shown







Popular posts from this blog

Список кардиналов, возведённых папой римским Каликстом III

Deduzione

Mysql.sock missing - “Can't connect to local MySQL server through socket”